Sie sind auf Seite 1von 59

INTERNAL

MEDICINE
PREVIOUS
BOARD EXAM
QUESTIONS

Prepared by
Julius Matthew M.Luzana

1
INTERNAL MEDICINE BOARD EXAM QUESTIONS
APMC BOARD EXAM
INTERNAL MEDICINE
2005

Name________________________________________ Score__________

A. Write the best answer in the space provided before each number.

_____1. Among the viruses associated with causing bronchiectasis, choose the main ones:
A. Coxsackie virus
B. Arena virus
C. Influenza Virus
D. Parvo virus

_____2. In patients with hospital acquired pneumonia, the following organisms are commonly found in
the ICU except:
A. Pseudomonas aeruginosa
B. Staphylococcus aureus
C. Acinetobacter spp.
D. Hemophilus influenzae

_____3. In patients with asbestosis, smoking greatly increases the incidence of


A. Lung Cancer
B. Mesothelioma
C. Pulmonary fibrosis
D. A and C

_____4. In the treatment of asthma, the following are classified as leukotriene modifiers except:
A. Montelukast
B. Nedocromil
C. Zileuton
D. Zafirlukast

_____5. In evaluating a possible lesion on chest xray, which of the following radiologic procedure will
give the best information.
A. Magnetic resonance imaging
B. CT scan with contrast
C. Ultrasound of the thoracic area
D. Radionuclide scanning
_____6. Pulmonary infections is common among patients with cystic fibrosis, therapy should be
directed against which of the following organisms:
A. Pseudomonas aeruginosa
B. Streptococcus pneumoniae
C. Chlamydia pneumoniae
D. None of the above

_____7. The following are radiologic signs which are found in patients with pulmonary embolism:
A. Westermark’s sign
B. Hampton’s sign
C. Palla’s sign
D. All of the above

_____8. Aside from lung cancer, the other malignancy (cies) most commonly associated with pleural
effusion are:
A. Breast carcinoma

2
INTERNAL MEDICINE BOARD EXAM QUESTIONS
B. Hepatoma
C. Renal cell cancer
D. Colon cancer

_____9. Key features to describe primary alveolar hypoventilation includes:


A. Chronic Respiratory Acidosis
B. Frequent episodes of central hypopnea
C. Development of daytime somnolence, disturbed sleep and morning headaches
D. All of the above

_____10. What is the most common disease condition in patients waiting for a lung transplant?
A. COPD
B. Idiopathic pulmonary fibrosis
C. Cystic fibrosis
D. Primary Pulmonary hypertension

11. In an alcoholic who comes in the emergency room complaining of ophthalmoplegia, lateral rectus
palsy and ataxia, the differential diagnosis should include:
A. cerebrovascular accident
B. Korsakoff’s psychosis
C. Wernicke’s disease
APMC Board exam Internal Medicine 2005 -page 3

_____12. In patients with atheroembolic stroke, the treatment most shown to have benefits
included:
A. Heparin
B. Aspirin
C. Dipyridamole
D. A and B

_____13. Current accepted treatment of asymptomatic carotid disease includes:


A. Carotid endarterectomy
B. Conservative management with statins and aspirin
C. Balloon angioplasty with stenting
D. All of the above

_____14. Asking a patient what they ate for breakfast this morning is testing his:
A. Working memory
B. Episodic memory
C. Long term memory
D. Procedural memory

_____15. Alzheimer’s disease is characterized by the following except:


A. reduction of acetylcholine in the nucleus basalis of Meynert
B. formation of neuritic plaques composed of AL amyloid, proteoglycans
C. Maybe genetic in origin, involving the APP gene mutation in chromosome 21
D. May involve presenilin-1 and presenilin-2 in chromosome 14 and 1 respectively

_____16. Patients with post infectious GN have the following findings:


A. Depressed C3 level
B. Depressed C4 level
C. Elevated ASO titer
D. A and B

_____17. Patients with acute renal failure secondary to gentamicin has the following findings:

3
INTERNAL MEDICINE BOARD EXAM QUESTIONS
A. oliguric renal failure
B. elevated eosinophil count in the urine
C. presence of dirty brown coarse granular cast in the urine sediments
D. presence of red blood cell cast in the urine

_____18. The correction of hyperkalemia which involves actual excretion of potassium is achieved by
what method?
A. administration of calcium gluconate
B. administration of sodium bicarbonate
C. administration of d50 water plus insulin
D. administration of kayaxelate

_____19. Kidney biopsy of a patient with membranous glomerulopathy will show:


A. Mesangial deposits
B. Subendothelial deposits
C. Subepithelial deposits
D. IgG linear antibodies

_____20. Patients with kidney transplantation who have received anti-lymphocyte antibodies (OKT3)
are prone to developing what type of infection in the immediate post transplant period?
A. Hepatitis B infection
B. Hepatitis C infection
C. Cytomegalovirus infection
D. None of the above

_____21. Patients who are undergoing dialysis for the 1 st time will experience this condition due to
rapid removal of toxic metabolites:
A. First use syndrome
B. Disequilibrium syndrome
C. Anaphylactic shock
D. Recirculation

_____22. Which of the following substances below are potent inhibitors of stone formation?
A. Uric acid
B. Inorganic pyrophosphate
C. Oxalate
D. None of the above

_____23. A 5 year old male presenting with multiple cysts in the 2 kidneys associated with hepatic
fibrosis is most likely to have:
A. autosomal dominant polycystic kidney disease
B. medullary cystic kidney disease
C. autosomal recessive polycystic kidney disease
D. medullary sponge kidney

_____24. Patient was complaining of reducing urine output after he started taking rifampicin and
isoniazid for pulmonary tuberculosis. The lab test shows normal sgpt but creatinine is 3.5mg/dl
(baseline of 1.0mg/dl at start of treatment). Urinalysis shows eosinophilia with WBC cast. What is
the expected etiology of the renal failure?
A. Acute tubular necrosis
B. Acute Glomerulonephritis
C. Acute Interstitial nephritis
D. Acute Vascular thromobosis

4
INTERNAL MEDICINE BOARD EXAM QUESTIONS
_____25. Which of the following post transplant malignancy is/are common?
A. Lung cancer
B. Breast Cancer
C. Colon cancer
D. Non-Hodgkins lymphoma

_____26. Which of the following physical findings suggest left ventricular heart failure?
A. Pulsus paradoxus
B. Pulsus alternans
C. Bisferiens pulse
D. Pulsus tardus

_____27. Which of the following statement is true of right bundle bunch block?
A. Only occur in pathologic conditions, that is in coronary artery disease.
B. May occur in subjects without structural heart conditions
C. May occur in patients with atrial septal defect
D. B and C

_____28. Which of the following imaging procedures is best to detect “hibernating myocardium”
A. Thallium 201 stress testing
B. Adenosine stress echo
C. Positron emission tomography with flurodeoxyglucose
D. Technetium 99m nuclear imaging

_____29. In patients with severe knee osteoarthritis and chronic obstructive lung disease, the best
way to test for myocardial ischemia is to do:
A. Adenosine stress echo
B. Dipyridamole stress echo
C. Dobutamine stress echo
D. None of the above

_____30.Which of the following conditions can cause AV nodal dysfunction?


A. Lyme’s disease
B. Sarcoidosis
C. Digitalis intoxication
D. All of the above

____31. A Cardiac Pacemaker is absolutely indicated in the following condition:


A. 3rd degree AV block
B. 2nd degree AV block Mobitz type I
C. 2nd degree AV block Mobitz type II following anteroseptal wall infarction
D. A and C

_____32. The method with the best chance to terminate Wolf-Parkinson-White syndrome is:
A. Digitalis
B. Verapamil
C. Beta-blocker
D. Catheter ablation

_____33. In a patient with abdominal aortic aneurysm, what diameter of the aortic aneurysm is
surgery absolutely indicated?
A. 4 cm
B. 4.5 cm

5
INTERNAL MEDICINE BOARD EXAM QUESTIONS
C. 5 cm
D. 5.5 cm

_____34. In treating deep vein thrombosis, the following statements are true:
A. Anticoagulation should be continued for 1 to 3 months
B. INR should be maintained between 2.5 to 3.5
C. Low molecular weight heparin should be started first then overlapped with warfarin
D. all of the above

_____35. In patients about to receive fibrinolytic therapy, what conditions are contraindicated to its
use:
A. Previous administration of streptokinase
B. Recent history of surgical procedure within the last 2 weeks
C. Patient taking anticoagulant with INR >2.0
D. All of the above

_____ 36. In a patient presenting with upper respiratory tract infection, the sudden onset of chest
pain with pericardial friction rub and heart failure is usually due to:
A. Hepatitis B virus
B. Herpesviruses
C. Coxsackievirus
D. Measles virus

_____37. In patients with suspected dengue hemorrhagic fever, the following should be done:
A. Infusion of crystalloid or colloid to prevent hemoconcentration
B. Administer steroid to prevent hypotension
C. Blood transfusion if the hematocrit goes below 36%
D. Prophylactic platelet transfusion if platelet count goes below 100,000

_____38. Which of the following viral agent does not cause hemorrhagic fever:
A. Dengue virus
B. Ebola virus
C. Hantavirus
D. HTLV I

_____39. Deeply Invasive Candidiasis is treated best with:


A. IV amphotericin B
B. Oral Fluconazole
C. Itraconazole
D. IV Caspofungin

_____40. The following are zoonotic infections, EXCEPT:


A. Brucellosis
B. Leptospirosis
C. Filariasis
D. Toxoplasmosis

_____41. The following are human infections caused by bacteria located intracellularly, EXCEPT:
A. Legionella infections
B. Chlamydia infections
C. Salmonella infections
D. Streptococcal infections

6
INTERNAL MEDICINE BOARD EXAM QUESTIONS
_____42. The drug of choice for methicillin-resistant Staphylococcus aureus is:
A. Carbapenems
B. Oxacillin
C. Fluoroquinolones
D. Vancomycin

_____43. Which of the following antibacterial agents does not penetrate cells well
A. Ampicillin
B. Macrolide
C. Metronidazole
D. Quinolones

_____44. Which antimycobacterial agent should be stopped if the patient develops gouty arthritis
A. Rifampicin
B. Isoniazid
C. Ethambutol
D. Pyrazinamide

_____45. Which vaccine is safe for use in pregnant women:


A. Hepatitis B
B. Measles
C. Mumps
D. Varicella

_____46. Very distinctive, pruritic small wheals that are surrounded by large areas of erythema
sometimes precipitated by fever or hot baths, are seen in:
A. Cholinergic Urticaria
B. Light urticaria
C. Vibratory angioedema
D. Aquagenic pruritus

_____47. Differential diagnosis of localized hypomelanosis includes:


A. Tinea Versicolor
B. Vitiligo
C. Nevus Depigmentosus
D. All of the above

_____48. Nikolsky’s sign which is separation of the epidermis is seen in:


A. Pemphigus Vulgaris
B. Toxic Epidermal Necrolysis
C. Stevens-Johnson Syndrome
D. All of the above

_____49. Tissue and blood eosinophilia are prominent features of:


A. Wegeners Granulomatosis
B. Allergic Granulomatosis
C. Polyarteritis Nodosa
D. Lymphomatoid Granulomatosis

_____50. A transient, pruritic linear wheal with a flare at a site in which the skin is briskly stroked
with a firm object is known as:
A. Dermatosis
B. Dermographics
C. Dermatography

7
INTERNAL MEDICINE BOARD EXAM QUESTIONS
D. Dermographism

_____51. The following are included in the criteria for the classification of Systemic Lupus
Erythematosus except:
A. Butterfly rash
B. Alopecia
C. Photosensitivity
D. Serositis

_____52. The following are true of systemic sclerosis:


A. Acquired Disease
B. Non-contagious multisystem disease
C. Raynaud’s phenomenon is the most common symptoms
D. All of the above

_____53. The following are features of Behcet’s disease except:


A. Multiple oral aphthous ulcers
B. Uveitis
C. Malar rash
D. None of the above

_____54. The following are true of Takayasu’s arteritis:


A. Chronic vasculitis of the aorta and its branches
B. Most common in young women of Asian descent
C. Biopsy is rarely done because of the vessel size involved
D. All of the above

_____55. Antiphospholipid syndrome:


A. Recurrent arterial or venous thrombosis
B. 2nd & 3rd trimester fetal death
C. (+) lupus anticoagulant
D. all of the above

_____56. Sjogren’s syndrome is characterized by the following except:


A. Dry eyes
B . Dry mouth
C. Dry hair
D. none of the above

_____57. The following are tumors commonly associated with dermatomyositis:


A. Melanoma
B. Lung cancer
C. Chronic lymphocytic leukemia
D. Pancreatic cancer

_____58. What is true of Rheumatoid Factor?


A. Can be found in patients with SLE and Sjogrens syndrome
B. Presence of Rheumatoid factor establishes the diagnosis of Rheumatoid arthritis
C. High titers and low titers of Rheumatoid factor has the same significance
D. All of the above

_____59. Which of the following patients is the LEAST suspicious of SLE:


A. 60 y.o female with fatigue, anorexia, cough, fever, increased ESR
B. 40 y.o female with oral ulcers, rash over cheeks, anemia, thrombocytopenia increased ESR,
arthritis

8
INTERNAL MEDICINE BOARD EXAM QUESTIONS
C. 30 y.o male with anemia, leucopenia, casts in the urine, decreased C3, headache, pleural effusion
D. 42 y.o female with skin rash, photosensitivity, anemia, fatigue

_____60. HLA B-27 is associated with what condition?


A. Ankylosing Spondylitis
B. Systemic Lupus Erythematosus
C. Sjogren’s syndrome
D. Scleroderma

_____61. The most life threatening side effect of cytotoxic chemotherapy is


A. Nausea and vomiting
B. Leucopenia
C. Alopecia
D. Mucositis

_____62. The following are goals of cancer treatment except:


A. To cure or control the disease whenever possible
B. To palliate symptoms in advanced disease
C. To increase over-all survival in terminally ill patients
D. To ensure that minimal functional impairment results from treatment

_____63. Telomerase, a protein found at the distal ends of DNA strands, normally fall off. However,
if it does not fall off, one of the following occurs:
A. Production of Immortal cells
B. Diseases of aging
C. Point mutation
D. Spontaneous regression

_____64. What is true of apoptosis


A. Programmed death of cells that lose normal growth regulations
B. Extrinsic pathway is activated by cross-linking members of the tumor necrosis receptor
superfamily
C. Intrinsic pathway is initiated by the release of cytochrome C and second mitochondrial activator
of caspases in response to a variety of noxious stimuli
D. All of the above

_____65. Current principles in antiangiogenic therapy depend on the following mechanism to halt
formation of new blood vessels in tumor growths:
A. Necrosis
B. Apoptosis
C. Anoikis
D. None of the above

_____66. Which of the following treatment may reverse the cytopenia in patients with advanced
Idiopathic myelofibrosis?
A. Hydroxyurea
B. Interferon
C. Splenectomy
D. Plateletpheresis

_____67. The enzyme that promotes myeloid proliferation in chronic myelogenous leukemia is:
A. thymidine kinase
B. tyrosine kinase
C. leucocyte alkaline phosphatase
D. none of the above

9
INTERNAL MEDICINE BOARD EXAM QUESTIONS
_____68. Fever in neutropenic patients is treated with empiric antibiotics against:
A. Staphylococcus
B. Gram positive aerobes
C. Gram negative aerobes
D. All

_____69. The differentiating agent that do not produce DIC and promotes promyelocytic maturation
in AML M3:
A. All-cis retinoic acid
B. All-trans retinoic acid
C. Tretinoin
D. Vitamin A

_____70.Which biochemical abnormality is associated with multiple myeloma?


A. Hypercalcemia
B. Hypocalcemia
C. Hypophosphatemia
D. None of the above

_____71. In a patient with hepatic cirrhosis, hepatic encephalopathy can be precipitated by all of the
following except:
A. Gastrointestinal bleeding
B. Metabolic Acidosis
C. Infection
D. Vomiting

_____72. Which of the following drug used for treating peptic ulcer disease should be avoided in
patients with renal failure
A. Sucralfate
B. Famotidine
C. Aluminum Hydroxide-Magnesium Hydroxide antacid
D. A and C

_____73. What can be said of type A gastric ulcer?


A. Also called autoimmune gastritis
B. Associated with Helicobacter Pylori infection
C. Associated with pernicious anemia
D. A and C

_____74. The most common organism isolated from the ascitic fluid of patients with spontaneous
bacterial peritonitis is:
A. Streptococcus pneumoniae
B. Staphylococcus aureus
C. Escherichia Coli
D. Bacteroides Fragilis

_____75. The most serious pulmonary complication of pancreatitis is:


A. Pleural effusion
B. Acute respiratory distress syndrome
C. Pneumonia
D. Pulmonary embolism

_____76. The following is true of Hepatitis B infection:

10
INTERNAL MEDICINE BOARD EXAM QUESTIONS
A. Accounts for >50% of fulminant hepatitis, a sizable proportion of which is associated with
Hepatitis D infection
B. DNA virus
C. Risk of chronic infection is much greater in adults than in infants
D. A and B

_____77. A 71 y.o. woman with a history of coronary artery disease presents to her family physician
for a routine check up. The physician notices that she has lost 20 lbs since her last visit 6 months ago.
When questioned, she gives a history of intermittent periumbilical pain that always begins about 30min.
after eating and lasts for about 2 hours. She claims that the pain is more after a large meal so she
began to eat less out of fear of the pain. The most likely diagnosis is:
A. Pancreatitis
B. Cholecystitis
C. Small bowel obstruction
D. Intestinal Ischemia

_____78. A 48 y.o. man with a history of alcohol abuse presents to the emergency room vomiting
bright red blood. All of the following should be included in the differential diagnosis except:
A. Ruptured Esophageal varices
B. Esophageal Reflux secondary to a hiatal hernia
C. Boerhaave’s syndrome
D. Mallory Weiss Syndrome

_____79. The most common site for ischemic colitis to occur is:
A. Splenic Flexure
B. Cecum
C. Rectum
D. Sigmoid Colon

_____80. All of the factors listed below may play a role in the development of duodenal ulcer disease
except:
A. Tobacco smoking
B. Helicobacter Pylori infection
C. Spicy Food ingestion
D. Genetic Factors

_____81. The cell mediated type of response is characterized by:


A. T cell involvement
B. Usually responsible for host defense against virus and fungus
C. Type IV hypersensitivity type of reaction
D. All of the above

_____82. The smallest concentration of antibody in the serum is:


A. IgA
B. IgM
C. IgE
D. IgM

_____83. The complement system can do all the following except:


A. Activate phagocytes
B. Lyze the target cells
C. Activates chemotaxis
D. inhibits opsonization

11
INTERNAL MEDICINE BOARD EXAM QUESTIONS
_____84. Mast cell degranulation is due to:
A. Crosslinking of 2 IgE antibodies
B. Primary response to antigen
C. antibody production
D. T cell activation

_____85.What is true of systemic mastocytosis:


A. Result of a point mutation
B. +Dariers sign
C. Presence of hepatosplenomegaly
D. all of the above

_____86. The following are true of hyperosmolar nonketotic coma except:


A. serum osmolality more than 320
B. ph 7.4
C. Hyperglycemia
D. serum ketones more than 2:1

_____87. A 28 year old male with mild hypertension had controlled blood pressure, he suddenly
developed hypertensive crisis with an elevated 24 hr urine metanephrines, what is the most likely cause
of his condition
A. Renal Artery Stenosis
B. Cushings syndrome
C. Pheochromocytoma
D. none of the above

_____88. The most reliable thyroid function test for diagnosing primary hypothyroidism:
A. Thyrotropin
B. Thyroid Stimulating Hormone
C. Free T4
D. Thyroid Scan

_____89. Clinical Feature of Conn’s disease:


A. Hyperkalemia
B. Metabolic Alkalosis
C. Hyponatremia
D. Alternating Hypotension-Hypertension

_____90. If a young female patient is amerorrheic with galactorrhea, complaining of slight visual
problems, the blood test to check is:
A. LH, FSH
B. Prolactin
C. Dopamine
D. Estradiol

_____91. You suspect a patient to have acromegaly. To screen for this problem you check:
A. IGF 1 level
B. Random Growth Hormone
C. TSH and T4 levels
D. serum ACTH level

_____92. A patient presents with polyuria and polydipsia. Serum sodium is 145 meq/l with a dilute urine
with specific gravity of 1.005. Patient has normal response to water deprivation without any response to
exogenous AVP (vasopressin). The condition is consistent with:

12
INTERNAL MEDICINE BOARD EXAM QUESTIONS
A. Dipsogenic diabetes insipidus
B. Central diabetes insipidus
C. Nephrogenic diabetes insipidus
D. SIADH

_____93. A 45 y.o. male presents with infertility. On examination he was noted to have small firm
testes and gynecomastia. The following statements is true of the above condition:
A. Most likely due to a secondary cause like pituitary gland trauma
B. Rule out possibility of exogenous estrogen intake
C. basic defect is a chromosomal abnormality= 47 XXY
D. Biochemical tests will reveal: low testosterone and low LH/FSH levels

_____94. Clinical manifestations of Growth Hormone Excess includes the following except:
A. Prognatism
B. Anhydrosis
C. Hypertension
D. Glucose Intolerance

_____95. 32 y.o. female with high blood sugar was examined to have “moon facies”, truncal obesity with
thin extremities, easy bruisibility and abdominal striae. She has an excess of which hormone
A. ACTH
B. Growth Hormone
C. Cortisol
D. A and C

_____96. This syndrome is described as “an imposter has replaced your relative”:
A. Munchausen’s syndrome
B. Capgras syndrome
C. Delirium Tremens
D. Rogers Syndrome

_____97. The following drugs are used in the treatment of bipolar disorders:
A. Lithium carbonate
B. Valproic Acid
C. Carbamezepine
D. All of the above

_____98. The following is true of schizophrenia:


A. Affected by both genetic and environmental events
B. Drugs which activate dopaminergic activities are used to treat it.
C. There is gradual increase in cortical volume over time
D. None of the above

_____99. In the treatment of general anxiety disorders, what is true:


A. Benzodiazepines can be given for more than 4 weeks without problems
B. Buspirone takes more than 1 week to act but has no abuse potential
C. Over 80% of patients with general anxiety disorders have depression
D. B and C

_____100. Criteria for major depression includes:


A. Loss of energy every day
B. depressed mood daily for at least 1 week
C. Hypersomnia nearly every day
D. A and C

13
INTERNAL MEDICINE BOARD EXAM QUESTIONS
MCU-FDT MEDICAL FOUNDATION
College of Medicine
Department of Medicine

1. In the pathogenesis of fever, which of the following statements is NOT true?


A. Shunting of blood from periphery to internal organs occur to conserve heat
B. Presence of PGE2 in the brain elevates set point of body temperature
C. Exogenous and endogenous pyrogens are destroyed by phagocytes
D. Elevated cAMP induces the release of monoamine neurotransmitters
2. The sympathetic nervous system can increase heat conservation through:
A. Increased heart rate
B. Vasoconstriction
C. Increased skeletal muscle contraction
D. Resetting of thermostat level at hypothalamus
3. Patients with fever may also complain of myalgia and/or arthralgia. These can be due to:
A. Increased muscular tone C. Effect of interferon
B. Effect of peripheral PGE2 D. Effect of endotoxins

4. A 20 y/o male was brought to the ER due to muscle rigidity. He also manifest with
hallucination, pupil dilation and increased temperature with dry skin. The above
findings can be due to:
A. Amphetamine abuse C. Heat stroke
B. Extrapyramidal symptoms D. Tetany

5. Increased heat production in cases of thyrotoxicosis can be secondary to:


A. Chemical reaction of basal metabolism C. Chemical thermogenesis
B. Increased skeletal muscle tone D. Vasodilation

6. What component of the lipid profile is anti-atherogenic?


A. High density lipoprotein C. Total cholesterol
B. Low density lipoprote.in D. Triglycerides

7. Which of the following manifestation is not a major criterion for rheumatic fever?
A. Erythema multiforme - marginatum C. Subcutaneous nodules
B. Migratory polyarthritis D. Sydenham’s chorea & carditis

8. A run of 3 consecutive premature ventricular depolarizations is known as:


A. Asystole C. Ventricular fibrillation
B. Supraventricular tachycardia D. Ventricular tachycardia

9. What is the most common type of atrial septal defect?


A. Membranous type C. Ostium secundum-midseptally
B. Ostium primum-downn syndrome D. Sinus venosus

10. What is the most common cause of secondary hypertension in the general
population?
A. Cushing’s syndrome C. Primary aldosteronism
B. Pheochromocytoma D. Renal diseases

11. Which of the following statements regarding cardiac symptoms is false?


A. Chest discomfort and/or dyspnea that appear only during activity are
characteristic of heart disease
B. Many patients with heart disease may be asymptomatic
C. Patients with valvular stenosis may manifest heart failure symptoms
D. It is rare for asymptomatic cardiac patients to develop sudden death, acute myocardial
infarction or stroke- common
(p. 1301)

14
INTERNAL MEDICINE BOARD EXAM QUESTIONS
12. Which of the following best characterize the second heart sound?
A. During inspiration, there’s and increase blood flow to the left ventricle casing a
delay in aortic valve closure
B. Wide splitting of the second heart sound may be due to right bundle branch block
C. In pulmonary hypertension the second heart sound is soft
D. P2 is normally louder than A2 in the second left intercostal space
(p. 1307-1308)

13. Valvular abnormalities and chamber dilatation are best diagnosed by:
A. Electrocardiogram C. Echocardiography
B. Treadmill exercise testing D. Nuclear imaging techniques
(p. 1320)

14. The electrocardiogram of a patient who is hypertensive for almost 5 years would
show which of the following?
A. Tall left precordial R waves and deep right precordial S waves
B. Diminished voltages in the limb leads (aVL or aVR)
C. Right atrial abnormality absence of repolarization abnormalities (ST
depression and T-wave inversions)
(p. 1314)

15. Which is not a component of the metabolic syndrome?


A. Waist circumference of >40 inches in men
B. Triglycerides >150 mg/dL
C. LDL > 40 mg/dL
D. Blood pressure >130/>85 mmHg
(p. 1432)

16. A patient who shows on auscultation with an accentuated first heart sound,
opening snap and a mid-diastolic rumbling murmur at the apex is suffering from:
A. Mitral regurgitation- holosystolic apical murmur, thrill, s3
B. Aortic regurgitation- high pitched diastolic blowing murmur,
Austin flint murmur low pitched diastolic rumble
C. Mitral stenosis
D. Aortic stenosis- systolic ejection murmur, soft single s2, s4

17. The most common etiology for secondary hypertension is:


A. Renal parenchymal hypertension C. Primary aldosteronism
B. Renovascular hypertension D. Pheochromocytoma
(p. 1464)

18. Liebman-Sacks lesion which is a small verrucous vegetation usually found in the
ventricular surface of the mitral valve is associated with this disease:
A. Rheumatoid arthritis C. Rheumatic Heart Disease
B. SLE D. Rheumatomyositis

19. Spironolactone is the drug of choice in the medical treatment of hypertension o


which of the following diseases?
A. Essential hypertension C. Phrochromocytone
B. Renal artery stenosis D. Hyperaldosteronism- (+)hypoK

20. Saw tooth appearance of EKG tracing is diagnostic of:


A. Ventricular tachycardia C. Atrial flutter
B. Ventricular fibrillation D. Atrial fibrillation

21. MVP has the following features EXCEPT: Barlow syndrome, floppy valve
A. More common in males- female
B. Most common course of isolated severe MR in North America
C. May cause syncope

15
INTERNAL MEDICINE BOARD EXAM QUESTIONS
D. Mid to late systolic click is the most important finding
Reference: Harrison’s Principles of Internal Medicine 15 th ed. pp. 1348

22. Which of the following is a major criterion in the Framingham Criteria for the
diagnosis of CHF?
A. Tachycardia C. Cardiomegaly
B. Dyspnea or exertion D. Extremity edema
Reference: Harrison’s Principles of Internal Medicine 15 th ed. pp. 1323

23. The most common primary cardiac tumor is:


A. Rhabdomyoma C. Lymphoma
B. Myxoma D. Sarcoma-malignant

24. An effective drug for meningococcal chemoprophylaxis:


A. Cefotaxime C. Chloramphenicol
B. Ceftriaxone D. Ceftizoxime
(See p.854, 16th ed. of Harrison’s)

25. The etiologic agent of chancroid is:


A. Treponema pallidum C. Haemophilus ducreyl
B. Calymmatobacterium granulomatis D. Neisseria gonorrhea
(See p.771 Ulcerative Genital Lesions, 16th ed. Harrison’s)

26. Pyomyositis is usually due to:


A. Group A Streptococcus C. Staphylococcus aureus
B. Streptococcus pyogenes D. Clostridium perfringens
(See p.744 Myositis/Myonecrosis, 16th ed. Harrison’s)

27. Cytoplasmic inclusion bodies found in certain neurons in the brain, and are
diagnostic of rabies:
A. Negri bodies C. Schuffner’s dots
B. Owl’s eye bodies D. James stipplings
(See p.1157 Pathogenesis of Rabies, 16th ed. Harrison’s)

28. Most vaccine for adults can be given also to pregnant women EXCEPT:
A. Tetanus, diphtheria C. MMR, Varicelia
B. Pneumococcal, influenza D. Rabies, Hepatitis B
(See p.720 Use of Vaccines in Special Circumstances, 16 th ed. Harrison’s)

29. The single most important diagnostic test to request for a patient with clinical signs
and symptoms suggestive of PTB is:
A. CXR C. TB culture
B. PCR D. Direct microscopy
(Sep.960 AFB microscopy, 16th ed. Harrison’s)

30. Urethritis can be documented on the basis of:


A. Mucopurulent or purulent discharge
B. Gram stain or urethral secretion > 5 wbcs/hpf
C. First void urine demonstrating > 10 wbcs/hpf
D. Either A or C
(See p.764 Approach to patient with suspected urethritis, 16 th ed. Harrison’s.
Letter B may be correct but remember that the gram stain is read per oil
immersion field)

31. Among the available test for Leptospirosis, the most sensitive and specific is:
A. Culture with EMJH
B. ELISA for antibodies
C. MAT using genus specific antigen for Leptospira patoc
D. Dipstick assay
(See p.990 Diagnosis of Leptospirosis, 16th ed. Harrison’s)

16
INTERNAL MEDICINE BOARD EXAM QUESTIONS
32. The following are true of malaria EXCEPT:
A. Hypoglycemia in falciparum malaria is associated with poor prognosis
B. Splenic enlargement in endemic areas reflects repeated infection
C. In severe malaria, one predicter of poor prognosis is the predominance of
immature P. falciparum parasites
D. The thick malarial smear has the advantage over the thin smear of
concentrating parasites, thus increasing diagnostic sensitivity
(See Table 195-3 p.1222, 16th ed. Harrison’s)

33. True of Staphylococcal infections EXCEPT:


A. Toxic Shock Syndrome is a life threatening condition caused by toxins
elaborated by S-aureus
B. Nikolsky’s sign is a feature of TSS – scalded skin syndrome
C. In Ritter’s disease, the skin often has a sandpaper-like texture and is tender
D. Staphylococcal pneumonia most commonly follows tracheal intubation of a
hospitalized patient or vital infection of the respiratory tract.
(See p.819 Staphylococcal Scalded Skin Syndrome, 16 th ed. Harrison’s)

34. The following are infection control measures EXCEPT:


A. Irrigation of urinary catheters, with or without antimicrobials, may
actually decrease infection risk
B. Replacing tubing circuits at intervals > 48 hours is one effective measure in
the aseptic care of respirator equipment
C. A control measure for surgical wound infection, aside from OR asepsis,
antimicrobial prophylaxis for high risk procedures
D. The recommended frequency of rotation of central venous catheter sites
is 5 days
(See p.778 every 72 to 96 hours, infections related to vascular access and
monitoring, 16th ed. Harrison’s)

35. Characteristics of the itch mite include the following:


A. It is an oval, ventrally flattened mite
B. It can live on beddings for 7 days
C. The female dies after copulation
D. The female lays 20 eggs per day
Andrews’ Diseases of the skin, 9th ed. p. 564-565

36. Infantile atopic dermatitis is characterized by the following:


A. Lesions are less exudative, drier and more papular
B. The buttocks and diaper area are often involved
C. Lesions are lichenified plaques
D. The face and scalp are more often involved
Andrews’ Diseases of the skin, 9th ed. p. 70

37. The main substance causing vasodilation in urticaria is:


A. Serotonin C. Bradykinin
B. Histamine D. Prostaglandins
th
Andrews’ Diseases of the skin, 9 ed. p. 163

38. Gutate psoriasis is usually preceded by:


A. Trauma C. Streptococcal pharyngitis
B. Hepatitis D.Viral hepatitis
Andrews’ Diseases of the skin, 9th ed. p. 221

39. An infectious disease transmitted by the body louse is:


A. Trench fever C. Tsutsugamushi fever

17
INTERNAL MEDICINE BOARD EXAM QUESTIONS
B. Dengue fever D. Scrub typhus
th
Andrews’ Diseases of the skin, 9 ed. p. 552-553

40. Hormone implicated in the pathogenesis of androgenetic alopecia.


A. Estrogen C. Testosterone
B. Cortisol D. Growth hormone
Andrews’ Diseases of the skin, 9th ed. p. 947-948

41. The 4 cutaneous findings of 11 criteria for diagnosis of SLE are:


A. Malar erythema, urticaria, photosensitivity and discoid LE
B. Malar erythema, photosensitivity, discoid LE, oral ulcers
C. Malar erythema, photosensitivity, skin erosion, oral ulcers
D. None of the above
Andrews’ Diseases of the skin, 9th ed. p. 180-181

42. The prothrombotic effect of coumarin (warfarin) during the early phases of its
administration is due to a rapid drop in the plasma concentration of:
A. Antithrombin C. Factor VII
B. Protein C D. Plasminogen

43. A 55-year old woman with Grave’s disease developed pallor and scleral jaundice.
Her peripheral blood showed pancytopenia, presence of macro-ovalocytes and
hypersegmented neutrophils. What is the most likely diagnosis?
A. Megaloblastic anemia C. Aplastic anemia
B. Myelodysplastic syndrome D. Paroxysmal nocturnal hemoglobinuria

44. A 55-year old man previously diagnosed to have stage 0 CLL developed anemia
and unconjugated hyperbilirubinemia. What is the single most important
laboratory test required to diagnose the cause of the anemia?
A. A repeat bone marrow examination C. Serum ferritin determination
B. Coomb’s test D. Serum folate and vitamin B12
determination
45. A 24-year old female suddenly developed jaundice, confusion, petechiae, and
fever. Peripheral blood examination showed anemia and thrombocytopenia.
Fragmented red cells were seen with nucleated red blood cells. Prothrombin and
partial thromboplastin time were normal. What is the treatment of choice for this
patient?
A. Plasmapharesis C. Pulse therapy with cyclophosphamide
B. High dose glucocorticoids D. Splenectomy

46. The treatment of choice in patients with acute promyelocytic leukemia.


A. Anthracycline + cytosine arabinoside
B. Vincristine + prednisone
C. All-transretinoic acid (ATRA) + anthracycline
D. Melphalan + prednisone

47. A 36-year old male was refereed to an internist after a routine blood examination
showed a low mean cell volume (MCV) but with no evidence of anemia. Stool is
negative for occult blood nor was there exposure to lead. Serum ferritin is normal.
What is the most likely diagnosis?
A. Early iron deficiency anemia C. Thalassemia minor
B. Anemia of chronic disease D. Sideroblastic anemia

48. A 32-year old male had excessive bleeding after a dental extraction. He has no
other medical problems and physical examination is unremarkable. His mother
has history of excessive bleeding. Bleeding time and PTT was prolong twice the
normal range but the prothrombin time is normal. What is the most likely
diagnosis?

18
INTERNAL MEDICINE BOARD EXAM QUESTIONS
A. Hemophilia A C. Factor XII deficiency
B. Hemophilia B D. von Willebrand’s disease

49. A 64-year old woman for preoperative evaluation was noted to have a markedly
prolong PTT. Her prothrombin time, platelet count, and bleeding time were
normal. She required blood transfusion blood transfusion after a dental extraction
2 years ago. Two siblings male and female has history of bleeding. The most
likely diagnosis is a deficiency of:
A. Factor VII C. Factor XI
B. Factor VIII D. Factor XII

50. An 18-year old male develops massive bleeding after removal of a large
sebaceous cysts. Prothrombin time, PTT, platelet count, and bleeding time were
normal. There is no family history of bleeding time. Which of the following tests
should be ordered?
A. Factor XI level C. Urea clot lysis test
B. Factor XII level D. Platelet aggregation study

51. A 59 year old, female, known hypertensive for 5 years, with poor compliance to
anti-hypertensive medications, now complains of pallor, easy fatigue, anorexia,
and itchiness. The kidneys on ultrasound appear contracted. Which casts will you
find in the urinalysis in this condition?
A. RBC casts C. Broad casts
B. WBC casts D. Hyaline casts
Harrison 16th ed. pp. 251

52. The leading cause of end stage kidney disease is:


A. Chronic glomerulonephritis C. Polycystic kidney disease
B. Diabetic nephropathy D. Chronic pyelonephritis
th
Harrison 16 ed. pp. 1688

53. A 40 year old, male, known diabetic for 8 years is noted to have +1 proteinuria.
The best medication at this time is:
A. Glibenclamide C. Rosiglitazone
B. Insulin D.Captopril
Harrison 16th ed. pp. 1689

54. A 25 year old, female, developed anasarca over 2 months period. Her urinalysis
showed RBC and RBC casts. Kidney biopsy revealed membranous
glomerulonephritis. This condition is most likely to be associated with:
A. Hepatitis B infection C. Diabetes mellitus
B. Hypertension D. Periarteritis nodosa
th
Harrison 16 ed. pp. 1687

55. A 44 year old, male, operated twice for kidney stones has an estimated creatinine
clearance of 40 ml/min. At what stage of renal failure is he in now?
A. Stage 1 C. Stage 3
B. Stage 2 D. Stage 4
Harrison 16th ed. pp. 1653

56. Dietary protein is restricted in which of these kidney conditions:


A. Acute Pyelonephritis C. Renal failure
B. Polycystic kidney disease D. Chronic glomerulonephritis
th
Harrison 16 ed. pp. 1652

57. A 25 year old, male, was admitted because of multiple gunshot wounds. On
admission, he was hypotensive and tachycardic. He was immediately scheduled
for exploratory laparotomy which lasted for about 4 hours. A total of 6 units of

19
INTERNAL MEDICINE BOARD EXAM QUESTIONS
blood were transfused. He was oliguric and acidotic, for the next 7 days. At what
phase of acute renal failure is he in?
A. Initiation phase C. Diuretic phase
B. Maintenance phase D. Recovery phase
Harrison 16th ed. pp. 1645

58. A 10 year old, boy, has puffy eyelids, tea-colored urine, and BP of 130 / 100,
noted after 2 weeks bout of sore throat and fever. The clinical diagnosis is
nephrotic syndrome. The condition that would initiate all the subsequent
components of the syndrome is:
A. Hypoproteinemia C. Lipiduria
B. Glomerular proteinuria D. Edema
Harrisson 16th ed. pp. 1684

59. A 44 year old, male, was admitted to Philippine Heart Center because of severe
chest pain. Initial findings indicate acute myocardial infarction. Within few minutes
after admission, he developed transient arrhythmia and drop in BP. The serum
creatinine went up from 0.9 mg% on admission to 1.3 mg% after 2 days. The urine
output was about 250-300 /day. These findings can be explained on the basis of:
A. Intratubular block by debris C. Back leak of glomerular filtrate
B. Renal hypoperfusion D. Nephrotoxicity from drugs
Harrison 16th ed. pp. 1645

60. A 33 year old, female, is suffering from SLE for the last 5 years. A year ago,
proteinuria and hematuria were noted in her urinalysis. And subsequently her
serum creatinine slowly deteriorated. The progression of her renal failure is due to:
A. Nephrotoxicity C. Renal hypoperfusion
B. Reduction in renal mass D. Interstitial nephritis
Harrison 16th ed. pp. 1653

61. One of the following is not a classification criteria in the diagnosis of Rheumatoid
arthritis:
A. Morning stiffness less than 1 hour
B. Radiographic change like erosion or decalcification of wrist bone
C. Serum rheumatoid factor positivity
D. Symmetric arthritis
Reference: Harrison’s Principles of Internal Medicine 14 th ed. pp. 1885

62. One of the following statements is not true with regards to the epidemiology of
SLE:
A. Common in women of child bearing age
B. Males are spared
C. More common in blacks than in white
D. Prevalence is form 15 to 50/100,000 population
Reference: Harrison’s Principles of Internal Medicine 14 th ed. pp. 1874

Aling Nena, 75-years old, female, 5’2 ft tall, 100 lbs. Consulted because of pain in the (R) knee
for a year; which is worst in the afternoon and after a long walk and improves with rest. On P.E. has
bony hypertrophy and worse crepitation.

63. The pain she is experiencing maybe due to any of the following EXCEPT:
A. Ligament insertion strain C. Muscle pain
B. Cartilage destruction D. Capsular stretching
Reference: Harrison’s Principles of Internal Medicine 14 th ed. pp. 1936-1937

64. One of the following statement about TB arthritis is true:


A. Culture of M. tuberculosis from the synovial biopsy is positive in 90% of cases
B. Joint destruction is rapid
C. Polyarticular involvement is more common
D. Treatment includes administration of Nafcillin and gentamycin

20
INTERNAL MEDICINE BOARD EXAM QUESTIONS
Reference: Harrison’s Principles of Internal Medicine 14 th ed. pp. 1947

65. A 23-years old female patient consulted because of on and off pains, malar rash,
oral ulcers and increasing hairloss. A probable diagnosis of SLE was made
confirmed by:
A. High ESR and (+) CRP
B. High titer dsDNA and hypocomplementemia
C. Leucopenia and anemia
D. Thrombocytosis
Reference: Harrison’s Principles of Internal Medicine 14 th ed. pp. 1877

Carla, 18-years old student is being treated for pneumonia. On the 4 th day of hospitalization
she noted swelling, redness and severe pain of the (L) knee. Aspiration of the knee revealed whitish,
purulent fluid consistent with septic arthritis.

66. The laboratory test you would request for the condition is:
A. Liver function test C. Culture and sensitivity of fluid
B. Urinalysis D. ANA
Reference: Harrison’s Principles of Internal Medicine 14 th ed. pp. 1945

67. The probability of a CPPD disease will reveal crystals appearing as:
A. Highly birefringent, large flat
B. Negatively birefringent, needle shape
C. Positively birefringent, rod shaped
D. Spheroidal aggregation
Reference: Harrison’s Principles of Internal Medicine 14 th ed. pp. 1942

68. The pain in osteoarthritis arise from the following structure EXCEPT:
A. Articular cartilage
B. Stretching of joint capsule
C. Stretching of periosteum covering osteophytes
D. Synovitis
Reference: Harrison’s Principles of Internal Medicine 14 th ed. pp. 1937

69. One of the following is not a classification criteria in the diagnosis of Rheumatoid
arthritis:
A. Morning stiffness less than 1 hour
B. Radiographic change like erosion or decalcification of wrist bone
C. Serum rheumatoid factor positivity
D. Symmetric arthritis
Reference: Harrison’s Principles of Internal Medicine 14 th ed. pp. 1885

70. One of the following statements is not true with regards to the epidemiology of
SLE:
A. Common in women of child bearing age
B. Males are spared
C. More common in blacks than in white
D. Prevalence is form 15 to 50/100,000 population
Reference: Harrison’s Principles of Internal Medicine 14 th ed. pp. 1874

Aling Nena, 75-years old, female, 5’2 ft tall, 100 lbs. Consulted because of pain in the (R) knee
for a year; which is worst in the afternoon and after a long walk and improves with rest. On P.E. as
bony hypertrophy and worse crepitation.

71. The pain she is experiencing maybe due to any of the following EXCEPT:
A. Ligament insertion strain C. Muscle pain
B. Cartilage destruction D. Capsular stretching
Reference: Harrison’s Principles of Internal Medicine 14 th ed. pp. 1936-1937

21
INTERNAL MEDICINE BOARD EXAM QUESTIONS
72. One of the following statement about TB arthritis is true:
A. Culture of M. tuberculosis from the synovial biopsy is positive in 90% of cases
B. Joint destruction is rapid
C. Polyarticular involvement is more common
D. Treatment includes administration of Nafcillin and gentamycin
Reference: Harrison’s Principles of Internal Medicine 14 th ed. pp. 1947

73. A 23-years old female patient consulted because of on and off pains, malar rash,
oral ulcers and increasing hairloss. A probable diagnosis of SLE was made
confirmed by:
A. High ESR and (+) CRP
B. High titer dsDNA and hypocomplementemia
C. Leucopenia and anemia
D. Thrombocytosis
Reference: Harrison’s Principles of Internal Medicine 14 th ed. pp. 1877

74. The recommended screening test for DM is:


A. FPG C. RBS
B. 2-hr OGTT D. HbAic
Reference: Harrison’s Principles of Internal Medicine 16 th ed. pp. 2154

75. All of the following symptoms are consistent with hypothyroidism EXCEPT:
A. Constipation C. Heat intolerance
B. Menorrhagia D. Hoarse voice
Reference: Harrison’s Principles of Internal Medicine 16 th ed. pp. 2109

76. The most specific feature of cushings syndrome is:


A. Centripetal obesity C. Hirsutism
B. Weight gain D. Hypertension
Reference: Harrison’s Principles of Internal Medicine 16 th ed. pp. 2139

77. All of the following chronic diabetic complications are microvascular in nature
EXCEPT:
A. Retinopathy C. Nephropathy
B. Neuropathy D. Peripheral vascular disease
Reference: Harrison’s Principles of Internal Medicine 16 th ed. pp. 2161

78. The following statements are true regarding the pathophysiologic abnormalities of
type 2 DM EXCEPT:
A. It is characterized by autoimmune destruction of the beta cells
B. There is impaired insulin secretion
C. Peripheral insulin resistance may be present
D. There is excessive hepatic glucose production
Reference: Harrison’s Principles of Internal Medicine 16 th ed. pp. 2157

79. The combination of insulin deficiency and hyperglycemia would result to the
following biochemical abnormalities EXCEPT:
A. Reduces the hepatic level of fructose 2-6 phosphate
B. Decreases the activity of pyruvate kinase
C. Promotes the process of glycogenolysis
D. Increases the levels of the GLUT4 glucose transporter
Reference: Harrison’s Principles of Internal Medicine 16 th ed. pp. 2159

80. The major effects of cortisol on body water are as follows EXCEPT:
A. Retards the migration of water into cells
B. Promotes renal water excretion

22
INTERNAL MEDICINE BOARD EXAM QUESTIONS
C. Stimulates vasopression secretions
D. Increases urine potassium excretion at high doses
Reference: Harrison’s Principles of Internal Medicine 16 th ed. pp. 2131

81. Choose the correct pair of tumor marker and cancer implicated:
A. Alphafetoprotein – colon cancer
B. CA-125 - ovarian cancer
C. Lactate dehydrogenase – myeloma
D. Carcinoembryonic antigen – hepatocellular carcinoma
Harrison’s Internal of Medicine, 16th ed. 439

82. The leading cause of cancer death in both men and women.
A. Lymphoma C. Lung CA
B. Nasopharyngeal CA D. Large cell CA
th
Harrison’s Internal of Medicine, 16 ed. 436, 506

83. The most common histologic subtype of lung cancer for the past 25 years.
A. Squamous or Epidermoid C. Small cell CA
B. Adenocarcinoma D. Large cell CA
th
Harrison’s Internal of Medicine, 16 ed. 506

84. Usually, these types of lung cancer have already spread at the time of
presentation and diagnosis, hence, surgery is unlikely to be curative. They are
managed primarily by chemotherapy with or without radiotherapy.
A. Squamous or Epidermoid C. Small cell CA
B. Adenocarcinoma D. Large cell CA
th
Harrison’s Internal of Medicine, 16 ed. 506

CASE: Leo Garcia, 68-year old male came in to your clinic because of urgency,
hesitancy and bloody urine: (Questions 167-169).

85. Prostate cancer is highly entertained, and the first screening modality that you will
perform is:
A. Digital rectal exam
B. Assay for serum PSA (prostate specific antigen)
C. Ultrasound of bladder/prostate
D. Transrectal ultrasound guided biopsy
Harrison’s Internal of Medicine, 16th ed. 446

86. Examinations revealed prostate cancer. The most common site for its metastasis:
A. Lungs C. Colon
B. Liver D. Bone
Harrison’s Internal of Medicine, 16th ed.

87. Deregulation of molecular mechanics controlling cell cycle progression is a


hallmark of cancer. Which phase of the cell cycle is critical in the regulation of cell
proliferation.
A. M C. S- dna synthesis
B. G1 D. G2
Harrison’s Internal of Medicine, 16th ed. 453
88. Certain human malignancies are associated with viruses. Correct examples are
the following, EXCEPT:
A. Burkitt’s Lymphoma – Epstein Barr virus
B. Cervical cancer – Human Papilloma virus 16,18,35
C. Hepatocellular carcinoma – Hepatitis B virus
D. Colon cancer – Helicobacter pylori
Harrison’s Internal of Medicine, 16th ed. 442

23
INTERNAL MEDICINE BOARD EXAM QUESTIONS
89. A woman with the following characteristics, has a greater risk of developing breast
cancer:
A. Menarche at 12 y/o, Primigravida at 18, menopause at 45
B. Menarche at 12 y/o, Nulligravida, menopause at 52
C. Menarche at 12 y/o, Multigravida, menopause at 45
D. Menarche at 16 y/o, Primigravida at 25, surgical menopause at 40 because of
TAHBSO
Harrison’s Internal of Medicine, 16th ed. 517

90. Tumor lysis syndrome is an oncologic emergency which may present as acute
renal failure. This is due to destruction of a large number of neoplastic cells
during chemotherapy. It is characterized by the following, EXCEPT:
A. Hyperuricemia C. Hypercalcemia
B. Hyperphosphatemia D. Hyperkalemia
th
Harrison’s Internal of Medicine, 16 ed. 581

CASE
A 55-year old male, farmer from Pampanga came to the ER because of difficulty of breathing.
Present condition started 2 weeks prior to consult as cough productive of opious grayish yellow
sputum accompanied by moderate grade fever body malaise and lost of apetite. Condition
progressed despite over the counter antitussive medication, until 1 day PTC he started to have
difficulty of breathing.

PH-smoker 1 pack of cigarette/day since age of 25 … no other serious illness in the past. Pertinent
P. E. Findings: RR – 28/min.
Chest: Symmetrical expansion: resonance of percussion, crackles on the left midlung to base
posteriorly with bilateral forced on expiration.

91. What disease conditions would you entertain?


A. Chronic bronchitis (COPD) C. Pneumonia
B. Mycosis D. All of the above

92. What diagnostic laboratory examination would you request for?


A. Chest x-ray C. CBC
B. Sputum gram stain, culture and sensitivity D. All of the above

93. How would you start your management?


A. O2 inhalation C. Nebulization
B. IV antibiotic D. All of the above

94. If arterial blood gas examination showed – ph7.33, pCO 2-50 mmHg, pCO2 of 75
mmHg and HCO3 of 28 mEq/L. What would be your interpretation?
A. Respiratory acidosis with hypoxemia- dec pH,inc pCO2
B. Respiratory alkalosis with hypoxemia
C. Respirator and metabolic acidosis
D. Adequate Oxygenation

95. What further laboratory examination would you add aside from above
examinations?
A. PFT C. ECG
B. Sputum for fungus D. A and B

96 - 100. A 60-year old male, 30-pack year smoker came in due to chronic cough and
mild exertional dyspnea for 3 years. He self medicated with mucolytic but

24
INTERNAL MEDICINE BOARD EXAM QUESTIONS
afforded no relief. On PE occasional wheezes and rhonchi were noted.

96. What is the most likely diagnosis:


A. Pulmonary Tuberculosis C. Chronic Obstructive Pulmonary Disease
B. Bronchial Asthma D. Pneumonia

97. What laboratory examination will you request to confirm the diagnosis:
A. Chest x-ray C. Chest CT scan
B. Sputum examination D. Spirometry

98. The mainstay in the treatment of this patient:


A. Inhaled bronchodilator C. Methylxanthines
B. Inhaled corticosteroids D. Antibiotics

99. A feature that best characterize chronic bronchitis.


A. Elastic recoil is severely decreased
B. Airway resistance is normal
C. Pulmonary hypertension moderate to severe at rest
D. Diffusing capacity is decreased

100. The most common mechanism of hypoxemia:


A. Decrease in inspired PO2 C. Shunt
B. Hypoventilation D. V/Q mismatching

101. A 40 year old male complains of chest pain, characterized by burning retrosternal
discomfort that radiates to the neck and angles of the jaw. It is aggravated by
bending forward, straining and lying recumbent and is worse after meals.This
classic symptom is called:
A. Heartburn C. Regurgitation
B. Angina D. Water Brash
p.1588 

102. Your clinical impression for this Patient is:


A. Acute myocardial infarction C. Gastric Ulcer
B. Gastroeophageal Reflux Disease D. Esophageal Carcinoma
p. 1592 

103. The pathophysiology of this disease is:


A. Spasm of the coronary artery
B. Increase acid secretion in the stomach
C. Decrease in the tone of the lower esophageal sphincter due to muscle
weakness
D. Exposure to carcinogens
p. 1592 

104. The drug of choice for this patient is:


A. Nitrates C. Calcium channel blockers
B. Antacids D. Protom pump inhibitors
p. 1593 
 
105. A 70 year old male with chronic constipation presented with sudden onset left
lower quadrant abdominal pain and high grade fever. Physical examination
showed direct and rebound tenderness at the left lower quadrant. Your clinical
diagnosis is:
A. Sigmoid Diverticulits C. Irritable Bowel Syndrome
B. Acute Appendicitis D. Colon Cancer
p. 1649 

106. Mark, a 7 yr old boy was passing by McDonald’s and smelled the aroma of
hamburgers. At this instant, acid secretion took place in his stomach. Which phase

25
INTERNAL MEDICINE BOARD EXAM QUESTIONS
of acid secretion was responsible for the acid secretion?
A. Cephalic C. Intestinal
B. Gastric D. Interdigestive

107. While Mark was eating his hamburger, the food in his stomach stimulated further
acid secretion. Which of the following substances is responsible for the increased
acid secretion?
A. Histamine C. Somatostatin
B. Gastrin D. Acetylcholine
C.
108. Mark grew up to become an executive of a leading company. Because of the
stresses of his job, he developed Duodenal Ulcers. The ulcers of Mark will be
found:
A. 3cm from the pyloric opening C. 2nd portion of the duodenum
B. 10cm from the duodenal bulb D. 3rd potion of the duodenum

109. Mark consulted his physician at MCU because of his Duodenal ulcer. The
physician examined Mark and found that Mark had a succussion splash. This
finding indicates that Mark had this DU complication.
A. Perforation C. Gastric outlet obstruction
B. Penetration D. Hemorrhage

110. Which of the following items below is a function of the pre-epithelial level of
defense to prevent mucosal damage?
A. The mucus gel functions as a unstirred water layer impeding diffusion of ions and
molecules including pepsin.
B. The surface epithelial cells act as ionic transporters that maintain intracellular pH and
bicarbonate production
C. Gastric epithelial cells bordering an injury migrate to restore a damage region
D. Epithelial cell renewal occurs and together with angiogenesis preserve integrity of the
gastric tissues

OUR LADY OF FATIMA UNIVERSITY


DEPARTMENT OF INTERNAL MEDICINE

1. Generally accepted indicator of the immunologic competence of the patient with HIV infection
A. level of plasma viremia
B. CD4+ T lymphocyte count
C. immunoglobulin level
D. PPD

2. The hallmark of HIV disease is


A. Kaposi’s sarcoma
B. opportunistic infections
C. cell-mediated immunodeficiency
D. humoral immunodeficiency

3. A 30 years old male job applicant consulted because of a hepatitis screening tests which
showed HBsAg(+), IgG anti-HBc(+), HBeAg(+), anti-HBs(-), anti-HBe(-). He has
A. Acute Hepatitis V viral infection
B. Chronic HBV infection, low infectivity
C. Chronic HBV infection high replication state
D. Cirrhosis

4. A medical technologist had accidental needlestick while at work. His chances of getting
infected is higher if the patient he handled is positive to HBeAg. What do you recommend?
A. Gamma globulin
B. Hepatitis B immune globulin
26
INTERNAL MEDICINE BOARD EXAM QUESTIONS
C. first dose of Hepatitis vaccine
D. interferon

5. A first year vet-med student was bitten by a dog he was examining, he claims to have had
anti-rabies immunization 1 year ago. What do you recommend?
A. Rabies vaccine booster on days 0 & 3
B. Give rabies immune globulin
C. wound treatment
D. all of the above

6. The earliest manifestation of Tetanus


A. Opisthotonus-3rd –arched back
B. dysphagia-2nd
C. lock jaw or trismus-increased tone of masseter muscle
D. muscle spasms-

7. Which of the following is a sign of severe (stage III) tetanus?


A. spasms lasting for less than 10 seconds-mild
B. lock jaw/trismus-dsphagia,rigidity,spasm- moderate
C. risus sardonicus- severe
D. localized muscle stiffness

8. The most frequent manifestation of typhoid fever is


A. Rose-spots
B. pulse-fever disproportion
C. prolonged persistent fever
D. splenomegaly

9. A social worker frequently assigned to areas of calamity consulted for prophylaxis against
Typhoid. She claims to have immunization with one injection of Vi polysaccharide vaccine 5
years ago. What do you recommend?
A. live attenuated vaccine 1 capsule as a booster
B. 3 doses of live attenuated oral typhoid vaccine
C. gamma globulin
D. none of the above
10. The following contribute to the renal failure in leptospirosis except
A. hypovolemia
B. acute tubular necrosis
C. acute cortical necrosis
D. direct renal tissue injury by the leptospires

11.A 40 y/o woman presented with 2 days diarrhea with fever and headache.
The stools were mucoid with blood, small volume but very frequent accompanied by
tenesmus. Fecalysis showed plenty of pus and red blood cells. The following may manifest
with the above type of diarrhea except
A. Shigella
B. E. coli
C. V. parahemolyticus
D. V. cholera

12. An OFW on vacation from his work in Africa had high fever and chills for 5 days. Stained
thick blood smears were reported to be positive to malaria. Which of the following is the
preferred treatment?
A. Chloroquine
B. Quinine
C. Mefloquine
D. Sulfadoxine/Pyrimethamine

13. The earliest physical manifestation of sepsis is


A. tachypnea
B. tachycardia
C. hypotension
D. altered mental status
27
INTERNAL MEDICINE BOARD EXAM QUESTIONS
14. The ff. plays a major role in the severe vasodilatation in septic shock
A. TNFa
B. nitric oxide
C. thromboxanes
D. Interleukin-1

15. Effect/s of activated Protein C that may counteract some of the mechanisms in sepsis
A. inhibits the coagulation cascade
B. increase fibrinolysis
C. inhibit leucocyte adherence to endothelium
D. all of the above

16. A 38y/o male who has had chemotherapy for lymphoma sought admission because of fever
& marked leukopenia. No focus of infection can be found. He was on Cefepime for several
days. What do you recommend?
A. Anti-anaerobic antibiotic
B. anti-fungal treatment
C. anti-pseudomonal antibiotic
D. vancomycin

17. A 30y/o male was admitted because of progressively severe abdominal pain. On laparotomy,
he was found to have ruptured appendix. Which of the following is appropriate?
A. Ticarcillin/Clavulanic
B. Piperacillin/Tazobactam
C. Imipenem
D. any of the above

18. The most common respiratory symptom of post primary tuberculosis


A. hemoptysis
B. chest pain
C. chronic cough
D. dyspnea

19. Impact of HIV infection on tuberculosis


A. more atypical cases of tuberculosis
B. more frequent occurrence of extrapulmonary TB
C. more difficult to identify because of similarities of symptoms
D. all of the above

20. Decreased alveolar ventilation results from the ff. condition, except
A. Decreased CNS drive
B. Decreased physiologic dead space
C. Neuromuscular diseases
E. Increased work of breathing with inadequate ventilation

21. The use of PEEP (positive end expiratory pressure) in mechanical


ventilation is helpful because it can
A. Limit venous return
B. Increase airway pressure
C. Decrease the cardiac output
D. Increase FRC and prevents alveolar collapse

22. Which of the ff. pathologic changes is NOT a characteristic of ARDS


A. Diffuse alveolar damage
B. Hyaline membrane formation
C. Hyperplasia of mucus gland and smooth muscle
D. Increased alveolar-capillary permeability

23. Superior vena cava syndrome is characterized by the following


A. Phrenic nerve paralysis and elevation of hemidiaphragm
B. Shoulder pain radiating to ulnar distribution of the arm
28
INTERNAL MEDICINE BOARD EXAM QUESTIONS
C. Edema and rubor of the face, neck and upper chest
D. Anhidrosis, miosis, ptosis of affected side- horner’s syndrome

24. One of the following is not a characteristic of the dyspnea of patients with COPD
A. variable, frequent at night-asthma
B. progressive & worsening over time
C. present daily
D. worse on exercise

25. A prominent cardiac silhouette on chest x-ray of a COPD with cor pulmonale patient is due to
A. Right ventricle-RVH
B. aorta
C. left atrium
D. left ventricle

26. The most common risk factor for development of ARDS is


A. pneumonia
B. sepsis
C. aspiration
D. severe trauma

27. Necrotizing pneumonia


A. cavities > 2 cm diameter
B. cavities < 2 cm diameter
C. process confined to alveoli contiguous to bronchi
D. interstitial involvement only

28. Pneumonia with erythema multiforme, hemolytic anemia, bullous myringitis


A. Streptococcus pneumoniae
B. Moraxella catarrhalis
C. Mycoplasma pneumoniae
D. Legionella pneumophilia

29. Risk factor for lung abscess


A. inhalation of bacteria
B. seizure
C. spinal anesthesia
D. amphetamine intake

30. Surgical indication for bronchiectasis


A. Cosmetic
B. severe dyspnea
C. massive hemoptysis
D. bilateral lung involvement

31. Usual source of pleural fluid in normal states


A. parietal pleura
B. visceral pleura
C. interstitium
D. peritoneal cavity

32. Most frequent cause of malignant pleural effusion


A. Lymphoma
B. breast Ca
C. bronchogenic Ca
D. mesothelioma

33. The most serious complication of hyperkalemia


A. seizure
B. cardiac toxicity- arrythmias
C. respiratory failure
29
INTERNAL MEDICINE BOARD EXAM QUESTIONS
D. muscle paralysis

34. Bence Jones Proteinuria is associated with


A. myeloma
B. hypertension
C. lymphoma
D. diabetes

35. The most common extrarenal system to develop cystic changes in ADPKD
A. liver
B. thyroid
C. pancreas
D. ovary

36. Urinary findings of patient with Tubulointerstitial Disease


A. Protein > 3 grams
B. hematuria
C. pyuria
D. RBC casts

37. Examination of the synovial fluid of the knee of a patient with osteoarthritis will reveal the
following except
A. Clear viscous fluid
B. Negative culture
C. WBC count of 200/hpf
D. (+) CPPD crystals

38. Inflammation of the 1st metatarsal joint due to MonoSodiumUrate crystal is called
A. Bursitis- Ca hydroxyapatite crystal
B. Podagra
C. Enthesitis – tendinous or ligamntous insertion on bones
D. Tophi- gout

39. The structure in the joint that is affected in osteoarthritis


A. Synovium-hyperuricemia
B. Capsule
C. Cartilage
D. Subchondral bone

40. One of the following is NOT a classification criteria in the diagnosis of Rheumatoid arthritis
A. Symmetric arthritis
B. Serum rheumatoid factor positivity
C. Radiographic change like erosion
D. Morning stiffness less than an hour- more than an hour

41. The earliest and consistent finding on x-ray of patient with ankylosing spondylitis:
A. Osteopenia
B. Osteophytes
C. Sacroilitis
D. Subchondral bone erosion

42. Crystal associated with pseudo gout


A. Monosodium urate
B. Calcium pyrophosphate dihydrate -chondroocalcinosis
C. Calcium oxalate
D. Uric acid

43. The most specific test in the diagnosis of SLE


A. ANA

30
INTERNAL MEDICINE BOARD EXAM QUESTIONS
B. ds DNA
C. Anti histone
D. Anti-sm

44. The major risk factor in the development of osteoarthritis


A. obesity
B. Aging
C. Trauma
D. Endocrine disorder

45. Bony hypertrophy in osteoarthritis of the distal interphalangeal joint is known as


A. Bouchard’s node- in nodal arthritis, hypertrophy of PIP
B. Boutonnieres deformity
C. Swan neck deformity- rheumatoid arthritis
D. Heberden’s node

46. Which of the following anemias is most likely to respond to the administration of
erythropoietin?
A. Iron deficiency anemia
B. Pernicious anemia
C. Pure red cell aplasia
D. Anemia of renal disease

47. Pancytopenia with an “empty” marrow is seen in


A. Aplastic anemia
B. Myelodysplastic syndrome
C. Paroxysmal nocturnal hemoglobinuria
D. Acute leukemia

48. The patient with intravascular hemolysis can present with all of the following clinical and
laboratory findings except
A. Low levels of serum haptoglobin
B. Increased indirect bilirubin and serum lactic dehydrogenase (LDH)
C. Splenomegaly
D. Hemoglobinuria and hemosiderinuria

49. The diagnosis of chronic ITP is established by


A. Marrow examination that shows increase megakaryocytes
B. Presence of anti-platelet-autobodies
C. Presence of giant platelets in the peripheral blood smear
D. Exclusion of other causes of thrombocytopenia

50. Which of the following is a strategy for primary prevention of cancer?


A. Paps smear
B. Digital rectal exam
C. Hepatitis B vaccination
D. Self-breast examination

51. Desired response to treatment if the goal of treatment is TO PALLIATE


A. Complete response
B. Partial response
C. Stable disease- increase life quality
D. Progressive disease

52. HER-2 neu is commonly expressed in which of the following malignancies?


A. Colonic Ca - k-ras
B. Breast Ca- CA 15-3,
C. Bronchogenic Ca- bcl-2
D. Endometrial CA

53. The most common malignant cause of mortality in both sexes is


31
INTERNAL MEDICINE BOARD EXAM QUESTIONS
A. Cervical Ca
B. Bronchogenic Ca
C. Breast Ca
D. Colonic Ca

54. A 20 y/o male was noted to be oliguria 2 days after hazing in fraternity. Urine was described
as reddish in color. Extreme tenderness noted all over the body. The cause of reddish urine
is
A. Hematuria
B. hemoglobinuria
C. myoglobinemia- increased in skeletal muscle injury(crushing injury,CO poisoning, electric burn)
D. porphyria

55. A 23y/o female complaining of 1 day duration of dysuria, with urgency & hesitancy, afebrile
with mild hypogastric tenderness. What is the most common uropathogen
A. E. coli- most common
B. Staph saprophyticus- common in sexually active women, normal flora of urin
C. Proteus
D. Candida

56. A 35y/o F, w/ Type 1 DM since 11y/o had a consultation for generalized edema. Had
hypertension for 2 years & had laser therapy of both eyes
for retinopathy. S crea is 3.8 mg% Hgb=80. The stage of diabetic glomerulopathy is
A. Initial Hyperfunctioning stage
B. Incipient Glomerulopathy
C. Overt Glomerulopathy
D. Azotemia Glomerulopathy
E. End Stage Glomerulopathy

57. 60 y/o male, smoker, had consultation for gross hematuria on & off for
3 months & weight loss. There is a palpable mass on (L) flank. The
laboratory examination which will be very helpful to clinically stage
this disease is
A. renal UTZ
B. IVP
C. CT scan with contrast
D. Renal scan

58. The most common glomerulopathy worldwide


A. Acute post strep GN
B. Lupus Nephritis
C. IgA Nephropathy
D. Thin Basement Membrane Disease

59. The most common functioning pituitary adenoma


A. Prolactinoma-galactorrhea,infertility
B. ACTH-secreting adenoma
C. TSH-secreting adenoma
D. Acromegaly

60. A 67 year old woman has just undergone near total thyroidectomy for
follicular thyroid carcinoma. Next step is
A. Chemotherapy with adriamycin
B. Radioactive iodine therapy- RAI tx
C. Levothyroxine replacement therapy
D. Observe and follow-up

61. Which of the following can differentiate a patient with Graves disease
from a patient with transient thyrotoxic phase of subacute thyroiditis
A. CTscan
B. Ultrasound
C. serum thyroid hormones level
D. I 131 scan of thyroid

32
INTERNAL MEDICINE BOARD EXAM QUESTIONS
62. Which of the ff. laboratory findings is consistent with Graves disease
A. High T4 with low TSH
B. High T4 with high TSH
C. low T4 with high TSH-thyroiditis
D. low T4 with low TSH

63. Diabetes is characterized by the following


A. Hyperglycemia
B. Disturbances in carbohydrate, fat and protein metabolism
C. Associated with absolute or relative deficiency of insulin
secretion and/ impaired action
D. All of the above

64. Dietary management of diabetic patient should include which of the ff.
A. Total caloric intake to achieve & maintain ideal body weight
B. Reduced intake of saturated fats and cholesterol
E. Avoidance of simple sugars
F. All of the above

65. Which of the following is diagnostic of diabetes


A. a single determination of FBS > 126 mg%
B. blood sugar of > 200mg% after an OGTT
C. RBS of > 200mg%
D. positive glucosuria

66. Which of the following is most diagnostic of hypercorticolism(cushing syndrome)


A. overnight dexamethhasone test
B. low dose dexamethasone test
C. Random serum cortisol determination
D. urinary cortisol determination

67. Which of the following will you consider in a young patient with hypertension with and
hypokalemia
A. Cushing’s disease
B. Conn’s disease
C. Pheochromocytoma-cathecholanine, most common HPH crisis,hyperCALcemia
D. Essential hypertension- most common cause of 1 HPN in adult

68. Cause of dysmotility type of chronic diarrhea


A. Hyperthyroidism
B. VIPoma
C. Radiation colitis
D. Lactase deficiency

69. True regarding Crohns disease


A. More common among non-smokers
B. Rectal bleeding is common
C. Surgery offers permanent cure
D. Peri-rectal fistulas are common

70. A surgically resected colon revealed tumor cells invading the


muscularis with 4 lymph nodes also (+)for tumor. Patient belongs to
Dukes stage:
A. A
B. B1
C. B2
D. C

71. Tenesmus is
A. Excessive passage of flatus
B. Indicative of sigmoid pathology
C. A symptom of inflammation of the anus
D. Painful straining & sensation of incomplete emptying during defecation

33
INTERNAL MEDICINE BOARD EXAM QUESTIONS
72. Best modality to determine the presence and level of intestinal
obstruction:
A. Colonoscopy
B. Plain film of the abdomen
C. Angiography
D. Gastroscopy

73. A 43/M had several episodes of bloody mucoid stools during the past 8 months. He had
received 2 courses of Metronidazole & 3 courses of
quinolones. Next most logical step is to
A. Do lower gut endoscopy
B. Repeat routine stool exam
C. Request for Sudan II staining
D. Request for fecal occult blood testing

74. Most common cause of acute pancreatitis


A. drugs and genetics
B. alcohol and gallstone
C. ERCP and hypercalcemia
D. Insect toxins and ischemia

75. Treatment for gallbladder stones seen incidentally by ultrasound on a normal individual
A. emergency cholecystectomy
B. elective cholecystectomy
C. ERCP
D. None of the above

76. Most common benign tumor of the esophagus


A. Polyps
B. Leimyoma
C. Hemangiomas
D. Squamous cell papilloma

77. Which of the following test for H. pylori cannot be used to monitor response to treatment
A. Rapid urease test
B. Serological test
C. Urea breath test
D. Culture

78. Which of the following food stuff is the most potent stimulant gastric
acid secretion:
A. Carbohydrates
B. Protein
C. Fats
D. Alcohol

79. The most common cause of obscure GI bleeding


A. Small bowel tumor
B. Dieulafoy’s lesion
C. Angiodysplasia
D. Arteriovenous malformation

80. The epithelial lining of the esophagus is


A. Cornified stratified squamous
B. Non cornified stratified squamous
C. Non cornified non stratified squamous
D. Cornified non stratified squamous

81. During the gap or window period, this is the only serologic evidence
Of hepatitis B
A. HBsAg
B. HBeAg

34
INTERNAL MEDICINE BOARD EXAM QUESTIONS
C. Anti-HBc
D. Anti-HBs

82. Despite persistence of virus in the liver, viral shedding in feces,


viremia and infectivity diminish rapidly once jaundice becomes apparent:
A. Viral hepatitis A
B. Viral hepatitis B
C. Viral Hepatitis C
D. Viral Hepatitis D

83. Which of the following drugs can relieve dyspnea in CHF?


A. Salbutamol by inhalation
B. Furosemide IV
C. Nitrates sublingual
D. Beta blocker per orem
E. Digitalis by IV route

84. Which of the drugs can dissolve the thrombus in acute coronary
A. Low molecular weight heparin
B. Unfractionated heparin
C. Aspirin
D. Streptokinase
E. Abciximab

85. Inspiration increases the intensity of loudness the murmur of


A. tricuspid regurgitation
B. atrial septal defect
C. MVP
D. Aortic stenosis- systolic ejection murmur

86. The most common primary malignant tumor of the heart is


A. atrial myxoma-benign
B. sarcoma
C. malignant
D. fibroma

87. All of the following are major Jones criteria for the diagnosis of rheumatic fever except
A. Chorea, subcutaneous nodule,migratory polyarthritis
B. erythema marginatum
C. fever- minor
D. carditis

88. True of Coarctation of Aorta


A. femoral pulse is weak and delayed
B. blood pressure in the leg is greater than the arm
C. primary cause of hypertension
D. presence of boot shaped heart on chest x-ray-tetralogy

89. Chest pain is present in patient with


A. aortic dissection
B. myocardial infarction
C. both
D. neither

90. Type of congenital heart disease that is acyanotic without a shunt


A. atrial septal defect
B. ventricular septal defect
C. coarctation of aorta
D. tetralogy of Fallot-cyanotic

91. Second heart sound is widely split and relatively fixed


A. atrial septal defect
B. ventricular septal defect
35
INTERNAL MEDICINE BOARD EXAM QUESTIONS
C. patent ductus arteriosus-continous machinery murmur
D. tetralogy of Fallot

92. Standing increases the intensity of the murmur of


A. tricuspid regurgitation
B. mitral stenosis- low pitched diastolic murmur @ apex
C. mitral valve prolapse-mid systolic murmur,opening snap
D. pulmonic stenosis

93. Chose the correct statement regarding aortic insufficiency


A. high pitched, blowing diastolic murmur
B. best heard with the bell of the stethoscope
C. decrease pulse pressure
D. best heard at the apex- mitral stenosis

94. A 28 y/o female patient came in with dyspnea. On auscultation the first heart sound is loud
and a low pitched diastolic murmur was noted at the apex. No LV heave was also noted. The
most likely condition is
A. aortic regurgitation-austin flint murmur (low diastolic rumble),blowing d murmur
B. mitral valve prolapse- mid systolic murmur
C. mitral stenosis
D. patent ductus arteriosus-continous machinery murmur
E. mitral regurgitation-holosystolic apical murmur

95. The severity of this component of tetralogy of Fallot determines the cyanotic manifestation of
this congenital anomaly
A. ventricular septal defect
B. obstruction to RV outflow-pulmonary valve stenosis
C. overriding aorta
D. RV hypetrophy
96. Infective endocarditis prophylaxis is indicated except
A. mitral stenosis
B. PDA
C. ASD, venosus type
D. MVP without MR

97. The most important physical sign of acute pericarditis


A. chest pain
B. pericaridal friction rub
C. paradoxical pulse
D. Kussmaul’s sign

98. Aspirin is indicated


A. acute myocardial infarction
B. acute pericarditis
C. both
D. neither

99. Most common cause of death in adults with diabetes


A. pericardial disease
B. coronary artery disease
C. valvular heart disease
D. renal failure

100. A 62 y/o man was admitted for evaluation of syncopal attack. He denied symptoms of
chest pain nor dyspnea on exertion. On P.E., a harsh grade IV/VI systolic ejection murmur
was heard at the right sternal border with radiation to the carotids. Carotid upstroke was
delayed. What is the most likely diagnosis?
A. pulmonic stenosis
B. mitral regurgitation
C. aortic stenosis
D. aortic regurgitation

36
INTERNAL MEDICINE BOARD EXAM QUESTIONS
DAVAO MEDICAL SCHOOL FOUNDATION
College of Medicine

INTERNAL MEDICINE

I. CHOOSE THE BEST ANSWER:

1. A 50 year old female presented with multiple erythematous


scaly papules on trunk and extremities. The following differential
diagnosis may be considered:
A. Pityriasis rosea
B. Psoriasis E
C. Pityriasis lichenoides chronica
D. Small plaque parapsoriasis
E. All of the above

2. On further examination, similar lesions appear on sites of even


trivial injuries give one diagnosis. B
A. Pityriasis rosea
B. Psoriasis
C. Pityriasis lichenoides chronica
D. Small plaque parapsoriasis
E. All of the above

3. The patient was treated with oral steroids with resolution of lesions. The steroid was
discontinued. Three days later, the patient became febrile followed by appearance of
erythematous patches on trunk and extremities with pinpoint pustules on the patches. What is
your diagnosis?
A. Mycosis Fungoides C
B. Drug reaction
C. Von Zumbusch
D. Folliculitis

4. The following drugs may be given in this patient except:


A. Retinoids
B. Methotrexate C
C. Steroids
D. Dapsone

5. Clinical clues to look for in patients with exfoliative dermatitis secondary to stasis
dermatitis:
A. Varicosities, edema,ulcers at the ankle A
B. Persistence of islands of normal skin within diffusely involved areas
C. Prominent white dermographism
D. Onycholysis and pterygium of the nails

6. Characteristic lesion of Mucha-Habermann disease:


A. Lichenoid papules
B. Macules with collarette scales
C. Papulonecrotic papules C
D. Silvery white scales
E. Purplish polygonal papules

7. One of the following is due to suprapapillary thinning of the


epidermis:
A. Koebner phenomenon
B. Woronoff ring D

37
INTERNAL MEDICINE BOARD EXAM QUESTIONS
C. Wickham’s striae
D. Auspitz sign
8. One of the following is due to focal epidermal thickening:
A. Koebner phenomenon
B. Woronoff ring C
C. Wickham’s striae
D. Button holing sign

9 The following diseases have unknown etiology except:


A. Psoriasis
B. Pityriasis rosea
C. Lichen planus E
D. Parapsoriasis
E. None of the above

10. Unusual distribution of lesions in patients with pityriasis rosea


include the following sites except:
A. Trunk
B. Face and head A
C. Scalp
D. Distal extremities

Reference: Andrews’ Diseases of the Skin 9th edition

11. The mechanism of hypoxemia in patients with an increased carbon dioxide and a
normal alveolar to arterial oxygen gradient is:
a. hypoventilation
b. diffusion impairment
c. ventilation perfusion mismatch
d. shunt

Harrison’s Principles of Internal Medicine, 15 th Edition, p.1452

12. According to the Philippine Clinical Practice Guidelines on CAP, the basis for the
diagnosis of pneumonia is established by:
a. when cough has been present for two weeks
b. clinical findings alone
c. radiologic diagnosis to confirm the diagnosis
d. by sputum G/S and C/S done routinely

Philippine Consensus Guidelines on Diagnosis and Management of Pneumonia

13. In the Gina (Global Initiative Against Asthma) Guidelines the addition of a long
acting beta agonist as controller medication is recommended in which category:
a. mild intermittent
b. mild persistent
c. moderate persistent
d. status asthmaticus

Global Initiative Against Asthma

14. A physiologic feature most suggestive of chronic bronchitis:


a. decreased DLCo
b. increased residual volume
c. increased airway resistance
d. decreased compliance

Harrison’s Principles of Internal Medicine, 15 th Edition

38
INTERNAL MEDICINE BOARD EXAM QUESTIONS
15. A clinical feature typical of emphysema:
a. cyanosis
b. bipedal edema
c. cachexia
d. recurrent exacerbations

Harrison’s Principles of Internal Medicine, 15 th Edition

16. Localized bronchiectasis is a feature of:


a. panhypogammaglobulinemia
b. primary ciliary dyskinesia
c. endobronchial obstruction
d. cystic fibrosis

Harrison’s Principles of Internal Medicine, 15 th Edition, p. 1486

17. Modality which has largely replaced bronchography for the diagnosis of
bronchiectasis:
a. PET (positron emission tomography)
b. fiberoptic bronchoscopy
c. HRCT (high-resolution computed tomography)
d. gallium Scan

Harrison’s Principles of Internal Medicine, 15 th Edition, p. 1486

18. The initial recommended procedure for the diagnosis of pulmonary


thromboembolism:
a. ventilation perfusion scan
b. leg ultrasound
c. d-dimer assay
d. pulmonary angiogram

Harrison’s Principles of Internal Medicine, 15 th Edition, p.1511

19. Antimicrobials for suspected atypical pathogens causing pneumonia EXCEPT:


a. macrolide or azalide
b. antipneumococcal fluroquinolone
c. tetracycline
d. beta lactam

Harrison’s Principles of Internal Medicine, 15 th Edition

20. The most common etiologic agent of pneumonia in HIV-infected patients with
CD4+ counts of <200/µL:
a. Mycobacterium avium-intracellulare
b. Cytomegalovirus
c. Pneumocystis carinii
d. Nocardia

Harrison’s Principles of Internal Medicine, 15 th Edition, p.1477

21. The single useful measurement for thyroid function in a suspect hypothyroid
patient is:
A. T3
B. T4
C. TSH
D. Thyroglobulin

39
INTERNAL MEDICINE BOARD EXAM QUESTIONS
Harrison’s Principle of Internal Medicine Vol. 2 15 th Edition page 2068
22. In large retrosternal goiter, when the arms are raised above the head, this cause the thyroid
mass to impinge on the blood vessels causing suffusion of the face, giddiness and syncope.
This is known as:
a. Basedow sign
b. Pembertons sign
c. Curtis sign
d. Reidel’s sign

Harrison’s Principle of Internal Medicine Vol. 2 15 th Edition page 2064

23. One of the following is a long glucocorticoid preparation.


a. Hydrocortisone
b. Prednisone
c. Bethamethasone
d. Triamcinolone

Harrison’s Principle of Internal Medicine Vol. 2 15 th Edition page 2104

24. The first to rise up among the lipoproteins in diabetes mellitus is:
a. Total cholesterol
b. LDL
c. HDL
d. Triglycerides

Harrison’s Principle of Internal Medicine Vol. 2 15 th Edition page 2124

25. The principal undesirable side effect of anti-thyroid drugs:


a. Wolf-Chaikoff effect
b. Agranulocytosis
c. Thyroid acropatchy
d. Fulminant hepatitis

Harrison’s Principle of Internal Medicine Vol. 2 15 th Edition page 2072

26. The single-thyroid drug which can be safely use in pregnant women:
a. Neomercazole
b. Thiamazole
c. Propylthiouracil
d. Tapazole

Harrison’s Principle of Internal Medicine Vol. 2 15 th Edition page 2073

27. The shortest acting oral hypoglycemics:


a. Glipizide
b. Glibenclamide
c. Gliclazide
d. Glimeperide

Harrison’s Principle of Internal Medicine Vol. 2 15 th Edition page 2133

28. Secondary hyperaldosterone has all of the following, EXCEPT:


a. Acidosis
b. Hypokalemia
c. Hypertension
d. Edema

Harrison’s Principle of Internal Medicine Vol. 2 15 th Edition page 2097

40
INTERNAL MEDICINE BOARD EXAM QUESTIONS
29. The other name of Multiple Neoplasia Type I is:
a. Lub syndrome
b. Stenon’s disease
c. Parry’s disease
d. Wermer’s syndrome

Harrison’s Principle of Internal Medicine Vol. 2 15 th Edition page 2184

30. The other term for Subacute thyroiditis are the following, EXCEPT:
a. Painless thyroiditis
b. Granulomatous thyroiditis
c. Giant cell thyroiditis
d. De Quervan’s thyroiditis

Harrison’s Principle of Internal Medicine Vol. 2 15 th Edition page 2074

31. One of these cardiac anti-arrythmic drugs contains 37% iodine and can induce thyrotoxicosis:
a. Flecainide
b. Verapamil
c. Amiodarone
d. Propranolol

Harrison’s Principle of Internal Medicine Vol. 2 15 th Edition page 2076

32. Conns’ syndrome commonly present with the following signs and symptoms, EXCEPT:
a. Hyperkalemia
b. Hypernatremia
c. Alkalosis
d. hypertension

Harrison’s Principle of Internal Medicine Vol. 2 15 th Edition page 2096

33. The effort less appearance of gastric or esophageal contents in the mouth is called:
a. Heartburn
b. Pyrosis
c. Vomiting
d. Regurgitation

Harrison’s Principle of Internal Medicine Vol. 2 13 th Edition page 1356 Chap 251

34. Nutcracker esophagus is associated with:


a. Achalasia
b. Scleroderma
c. Diffuse esophageal spasm
d. Esophageal CA

Harrison’s Principle of Internal Medicine Vol. 2 13 th Edition page 1358 Chap 251

35. The most common complication of Gastric Ulcer is:


a. Perforation
b. Penetration
c. Obstruction
d. Hemorrhage

Harrison’s Principle of Internal Medicine Vol. 2 13 th Edition page 1371 Chap 252

41
INTERNAL MEDICINE BOARD EXAM QUESTIONS
36. A 40 year old patient with a history of peptic ulcer surgery due to massive upper GI bleeding
came in because of lightheadedness, diaphoresis confusion 90 min after eating. Your
diagnosis is:
a. Dumping syndrome
b. Bile reflux gastritis
c. Afferent loop syndrome
d. Postvagomtomy diarrhea

Harrison’s Principle of Internal Medicine Vol. 2 13 th Edition page 1374 Chap 252

37. A 39 year old seaman showed you the following hepatitis serologic profile: HbsAg (+), IgM anti-
HAV (+), IgM anti-HBc (+), anti-HCV (-). Your diagnostic interpretations is:
a. Acute Hep A superimposed on Chronic Hep B
b. Acute Hep A and B
c. Acute Hep A
d. Acute Hep B

Harrison’s Principle of Internal Medicine Vol. 2 13 th Edition page 1470 Chap 266

38. Stage II hepatic Encephalopathy is characterized by:


a. Coma
b. Marked confusion, incoherent speech
c. Lethargy
d. Euphoria or depression

Harrison’s Principle of Internal Medicine Vol. 2 13 th Edition page 1494 Chap 269

39. A collection of tissue, fluid, debris, pancreatic enzymes and blood which develop over a period
of 1-4 weeks after the onset of acute pancreatitis is:
a. Phlegmon
b. Pseudocyst
c. Abscess
d. Pseudoaneurysm

Harrison’s Principle of Internal Medicine Vol. 2 13 th Edition page 1525 Chap 274

40. A sudden and severe loss of vision in patient with acute pancreatitis is:
a. Diabetic retinopathy
b. Glaucoma
c. Purstcher’s retinopathy
d. Retinal detachment

Harrison’s Principle of Internal Medicine Vol. 2 13 th Edition page 1356 Chap 274

41. The most consistently observed risk factor for development of pancreatic cancer is:
a. Chronic alcoholism
b. Diabetes mellitus
c. Cigarette smoking
d. Acute pancreatitis

Harrison’s Principle of Internal Medicine Vol. 2 13 th Edition page 1352 Chap 275

42. The following frequent signs and symptoms of pancreatic cancer, EXCEPT:
a. Abdominal pain
b. Weight loss
c. Jaundice
d. Splenomegaly

Harrison’s Principle of Internal Medicine Vol. 2 13 th Edition page 1532 Chap 275

43. A serum creatinine level starts to increase when the glomerular filtration rate is:

42
INTERNAL MEDICINE BOARD EXAM QUESTIONS
a. 75ml/min
b. 50ml/min
c. 25ml/min
d. 10mml/min

Harrison’s Textbook of Internal Medicine, 15th edition

44. Metabolic acidosis in moderate to severe renal failure is due to:


a. Increase acid production
b. Decreased acid excretion
c. Increase serum potassium
d. Nephron loss

Harrison’s Textbook of Internal Medicine, 15th edition

45. A source of endogenous toxin which can cause acute renal failure is:
a. Rhabdomyolysis
b. Sepsis
c. Radiocontrast agents
d. Antibiotics

Harrison’s Textbook of Internal Medicine, 15th edition

46. The most common complication of end stage renal disease is:
a. Congestive health failure
b. Hypertension
c. Recurrent infections
d. Gastrointestinal bleeding

Harrison’s Textbook of Internal Medicine, 15th edition

47. An absolute contraindications of kidney transplantation is:


a. Active glomerulonephritis
b. Presence of vesical or urethral abnormality
c. Iliofemoral occlusive disease
d. Psychiatric problem

Harrison’s Textbook of Internal Medicine, 15th edition

48. The major cause of death in end stage disease patients receiving dialysis is:
a. Bleeding
b. Infection
c. Uremia
d. Cardiovascular disease

Harrison’s Textbook of Internal Medicine, 15th edition

For nos. 49 – 52: A 68 year old female was admitted because of LBM. Her condition started 3 days
PTA as abdominal pain, followed by LBM characterized as watery, non-bloody, and occurring 4-5
times per day. LBM became more persistent and profuse, hence, the admission.
Her past history included mild hypertension of 2 years, which was treated irregularly, and joint
pains of 10 years.
On admission, vital signs were the following: BP = 90/60; HR = 100/min; RR = 24/min; T = 38.5
C. She was weak-looking but conscious. He r skin and tongue were dry. The heart had a gr 2/6
systolic aortic murmur. The abdomen was soft, tympanitic, and slightly tender all over, with
hyperactive bowel sounds.
Lab result revealed the following: CBC: Hb = 132g/l, WBC ct = 10.2, Seq. = 0.85, Ly = 0.15.
Urinalysis: pH = 5.0, SG = 1.030, protein = +, sugar = trace, pus cells = 1-3/hpf, rbc = 5-10/hpf,
hyaline cast = 5-10/hpf. FBS = 6.4 m mol/I; serum creatinine = 330 mmol/l; serum Na+ = 136 mmol/l;
serum K+ = 3.4 mmol/l. Stool exam: pus cell = 5-10/hpf; rbc = 3-5/hpf. ECG = sinus tachycardia.

43
INTERNAL MEDICINE BOARD EXAM QUESTIONS
49. What is the cause of the abnormal serum creatinine?
a. Acute renal failure
b. Chronic renal failure
c. Nephrosclerosis
d. Rapidly progressive renal failure

Harrison’s Textbook of Internal Medicine, 15th edition

50. What is the possible etiology?


a. Hypertension
b. Dehydration
c. Nephrotoxicity
d. Diabetic nephropathy

Harrison’s Textbook of Internal Medicine, 15th edition

51. If this is an uncomplicated acute renal failure, when would you expect the kidneys to recover?
a. 1-2 weeks
b. 2-3 weeks
c. 3-4 weeks
d. 2-3 months

Harrison’s Textbook of Internal Medicine, 15th edition

52. If this patient had chronic renal failure secondary to diabetic nephropathy when would you
expect end stage renal disease to set in?
a. 3-5 years
b. 5-10 years
c. 10-15 years
d. 15-20 years

Harrison’s Textbook of Internal Medicine, 15th edition

53. The primary factor that regulates erythropoietic activity is:


a. The kidney
b. Erythropoietin
c. Bone marrow
d. Oxygen

Harrison’s Textbook of Internal Medicine, 15th edition

54. Maximal iron absorption occurs in the:


a. Stomach
b. Duodenum and upper jejunum
c. Terminal ileum
d. Colon

Harrison’s Textbook of Internal Medicine, 15th edition

55. Phagocytosis is a primary function of:


a. Lymphocytes
b. Eosinophils
c. Neutrophils
d. Basophils

Harrison’s Textbook of Internal Medicine, 15th edition

56. The like hood that a daughter of a patient with severe hemophilia A will be a carried of
hemophilia is:
a. 0%

44
INTERNAL MEDICINE BOARD EXAM QUESTIONS
b. 25%
c. 50%
d. 100%

Harrison’s Textbook of Internal Medicine, 15th edition

57. Aspirin affects the platelets by:


a. Shortens platelets life span
b. Promotes platelets aggregation
c. Decreases platelet production
d. Prolongs the bleeding

Harrison’s Textbook of Internal Medicine, 15th edition

58. Which of the following hematopoietic growth factors is non-lineage specific and would induce
formation of progenitor cells.
a. Granulocyte colony stimulating factors
b. Macrophage colony stimulating factor
c. Granulocyte-macrophage colony stimulating factor
d. Erythropoietin

Harrison’s Textbook of Internal Medicine, 15th edition

59. One of the following is true in patients with New York Heart Association functional class IV:
a. Symptoms occur with ordinary activity
b. Asymptomatic at rest
c. Symptoms occur even at rest
d. None of the above

Harrison’s Textbook of Internal Medicine, 15th edition

60. Pulsus paradoxus may be seen in:


a. Pericardial tamponade
b. COPD
c. Superior vena cava obstruction
d. All of the above

Harrison’s Textbook of Internal Medicine, 15th edition

61. One or some of the following is/are true regarding jugular venous pulse (JVP), EXCEPT:
a. The left internal jugular vein is the best to use in JVP measurement
b. Cannon a waves may be seen in complete heart block
c. It reflects phasic pressure changes in the left atrium
d. A and C

Harrison’s Textbook of Internal Medicine, 15th edition

62. Prominent a waves of the JVP waves are seen in the following, EXCEPT:
a. Severe mitral stenosis with atrial fibrillation
b. Severe tricuspid stenosis in sinus rhythm
c. Ventricular tachycardia
d. A and B only

Harrison’s Textbook of Internal Medicine, 15th edition

63. The following are true regarding cardiac activation, EXCEPT:


a. At the rest the cardiac cell are polarized
b. The polarized state is brought about by the Na+-K-+ inhibiting pump

45
INTERNAL MEDICINE BOARD EXAM QUESTIONS
c. During phase 2 of the action potential there is slow inward current through the L-type
calcium channel
d. A and C only

Harrison’s Textbook of Internal Medicine, 15th edition

64. The following are components of Wolf-Parkinsons-White syndrome, EXCEPT:


a. Short PR interval
b. Delta wave
c. Paroxysmal ventricular tachycardia
d. All of the above

Harrison’s Textbook of Internal Medicine, 15th edition

65. The following are true regarding digitalis in heart failure, EXCEPT:
a. It is now obsolete since the discovery of ACE inhibitors which block the RAA axis
b. It is particularly effective in patient with systolic heart failure by stimulating myocardial
contraction
c. It improves ventricular emptying by improving myocardial contractility
d. It has little or no value in heart failure patients in sinus rhythm and diastolic dysfunction

Harrison’s Textbook of Internal Medicine, 15th edition

66. The following are true regarding murmurs, EXCEPT:


a. Tricuspid regurgitation murmurs are louder during expiration
b. With valsalva maneuver, the murmur of hypertrophic obstructive cardiomyopathy
increases.
c. With standing mitral regurgitation murmur decreases
d. A and C

Harrison’s Textbook of Internal Medicine, 15th edition

67. The following are true regarding contractile process of the cardiac muscle, EXCEPT:
a. During activation, the A band remains constant in length, whereas the I band shortens
and the Z lines move towards one another
b. The interactions between the actin and myosin is inhibited by the tropomyosin during
relaxed stage
c. Troponin I, accelerates cardiac contraction
d. A and B

Harrison’s Textbook of Internal Medicine, 15th edition

68. Continuous murmurs are heard in the following, EXCEPT:


a. Coarctation of the aorta
b. Ruptured coronary sinus of valsalva into the left ventricle
c. PDA with normal PA
d. Pregnant woman with mammary soufflé

Harrison’s Textbook of Internal Medicine, 15th edition

69. Which of the following exposures is most likely to result in transmission of HIV to an uninfected
person?
a. One use of injected drugs with a needle previously used by an HIV infected person
b. Once instance unprotected sexual intercourse with an HIV infected partner
c. being born to a woman with symptomatic HIV disease

46
INTERNAL MEDICINE BOARD EXAM QUESTIONS
d. sustaining a needle stick injury with a needle used to draw blood from a patient with
AIDS

Harrison’s Textbook of Internal Medicine, 15th edition

70. An HIV man presents with Pneumocystic pneumonia, oral hairy leukoplakia, oral candidiasis.
Select the stage of HIV disease.
a. Acute retroviral syndrome
b. Early disease (CD4 count greater than 500/ul)
c. Intermediate disease (CD4 count 200 to 500/ul)
d. Late disease (CD4 count 50 to 200/ul)

Harrison’s Textbook of Internal Medicine, 15th edition

71. A 26 year old female prostitute with recently diagnosed HIV infection has odynophagia and
vaginal discharge without fever. Select the most likely pathogen.
a. Candida
b. Cytomegalovirus
c. Hepatitis C
d. Microsporidia

Harrison’s Textbook of Internal Medicine, 15th edition

72. True statements about MDR (multidrug-resistant) tuberculosis:


a. It is more likely to cause extra pulmonary disease than is drug-susceptible TB
b. It is likely to present with cavitary pulmonary disease than is drug-susceptible TB
c. HIV-infected injection drug users have the highest risk of MDR-TB of all HIV (+) patient
d. Noncompliance with treatment for drug-susceptible TB is the most important
cause of this disease

Harrison’s Textbook of Internal Medicine, 15th edition

73. Which of the following statements is true about the bactericidal activity of antibiotics?
a. B lactam antibiotics are bactericidal in vivo at all concentrations
b. Only bactericidal antibiotics are effective in sterilizing abscesses
c. Relapse rates of endocarditis are higher when bacteriostatic antibiotics are used
in place of bactericidal drugs
d. The MIC of the antibiotic predicts its bactericidal activity

Harrison’s Textbook of Internal Medicine, 15th edition

74. Antibiotics prophylaxis is advised for prevention of endocarditis in all of the following
conditions, EXCEPT:
a. Previous episode of infectious endocarditis
b. Mitral valve prolapse with an audible murmur
c. Hypertrophic cardiomyopathy
d. Coronary artery disease after coronary bypass graft

Harrison’s Textbook of Internal Medicine, 15th edition

75. All of the following STD facilitate transmission of HIV, EXCEPT:


a. Human papilloma virus
b. Chancroid
c. Syphillic ulcers
d. Herpes simplex virus

Harrison’s Textbook of Internal Medicine, 15th edition

76. All of the following statements are true about human papilloma virus, EXCEPT:
a. An abnormal Pap test requires consideration of human papilloma virus

47
INTERNAL MEDICINE BOARD EXAM QUESTIONS
b. Women with some serotypes of human papilloma virus should be routinely screened for
cervical neoplasia
c. Cervical neoplasia in a 23 year old woman suggest HIV disease
d. Interferon alpha eliminates human papilloma infection

Harrison’s Textbook of Internal Medicine, 15th edition

77. No clinical evidence for “true” bacteremia. Select the most likely blood culture.
a. E.coli
b. Epidermidis
c. Clostridium welchi
d. S. Aureus

Harrison’s Textbook of Internal Medicine, 15th edition

78. Rheumatic heart disease. Select the organism most likely to cause endocarditis.
a. A. aureus
b. S. bovis
c. E.faecalis
d. Viridans streptococci

Harrison’s Textbook of Internal Medicine, 15th edition

79. A 63 year old man with diabetes and peripheral vascular disease. Select the most appropriate
prophylaxis.
a. Rabies vaccine
b. Pneumococcal vaccine
c. Hepatitis B vaccine
d. Meningococcal vaccine

Harrison’s Textbook of Internal Medicine, 15th edition

80. Approximately one third of patient infected with this virus may respond to treatment with
interferon alpha.
a. Norwalk virus
b. Rotavirus
c. Human herpesvirus 6
d. Hepatitis C virus

Harrison’s Textbook of Internal Medicine, 15th edition

81. The following are risk factor for the increasing prevalence of drug resistant TB, EXCEPT:
a. HIV-1 infection
b. Addition of a single new drug to a failing anti-TB regimen
c. Failure to complete treatment of active TB
d. Increased age

Harrison’s Textbook of Internal Medicine, 15th edition

82. True about granulocyte-colony stimulating factor, EXCEPT:


a. It reduces the number of febrile days and the frequency of infection in patients receiving
cytotoxic therapy for lung carcinoma
b. It stimulates the proliferation of neutrophils in the bone marrow
c. Its principal side effects is bone pain
d. Its use should be avoided in patient receiving treatment for myeloid leukemia

Harrison’s Textbook of Internal Medicine, 15th edition

83. True about coagulase-negative staphylococci (CNS), EXCEPT:

48
INTERNAL MEDICINE BOARD EXAM QUESTIONS
a. Most CNS are resistant to B lactam and therefore require a glycopeptide antibiotic in
initial, empiric, or definitive antibiotic regimens
b. CNS often cause true bacteremias, even in the absence of a foreign body
c. CNS strains frequently cause foreign body infections
d. Definitive therapy of CNS central nervous system shunt infections usually requires
shunt removal.

Harrison’s Textbook of Internal Medicine, 15th edition

84. This is characterized by an inability to recall important personal information, usually of a


traumatic or stressful nature:
a. Dissociative Amnesia
b. Dissociative Fugue
c. Depersonalization Disorder
d. Dissociative Identity Disorder

Kaplan & Saddock’s Synopsis of Psychiatry - page 678

85. Characterized by the presence of two or more distinct personality states:


a. Depersonalization Disorder
b. Dissociative Identity Disorder
c. Dissociative Amnesia
d. Dissociative Fugue

Kaplan & Saddock’s Synopsis of Psychiatry - page 681

86. The following are features seen in Kleptomania, except:


a. Increasing sense of tension immediately before committing the theft
b. Pleasure or relief at the time of committing the theft
c. Stealing is committed to express anger
d. Recurrent failure to resist impulses to steal objects

Kaplan & Saddock’s Synopsis of Psychiatry - page 786

87. Trichotillomania is associated with:


a. Fire setting
b. Gambling
c. Stealing
d. Pulling of hair

Kaplan & Saddock’s Synopsis of Psychiatry - page 790

88. The following are impulses control disorders, except:


a. Generalized anxiety disorder
b. Pathological gambling
c. Kleptomania
d. Intermittent explosive disorder

Kaplan & Saddock’s Synopsis of Psychiatry - page 783

89. The following are features of Anorexia Nervosa, except:


a. Refusal to maintain body weight at or above a minimally weight for age
b. Intense fear of gaining weight
c. Awareness of the seriousness of the current low body weight
d. Amenorrhea

Kaplan & Saddock’s Synopsis of Psychiatry - page 741

90. Features of Bulimia Nervosa, except:


a. Normal weight to slightly overweight or underweight

49
INTERNAL MEDICINE BOARD EXAM QUESTIONS
b. Poor sexual adjustments
c. Increased frequency of depressive symptoms
d. Binge eating

Kaplan & Saddock’s Synopsis of Psychiatry - page 748

91. A 60 year old male was brought to the emergency room when he was found to be wandering in
the vicinity, apparently lost and distressed. He was able to state his name, his daughter’s name, but
could not recall how he came to be lost. When his daughter arrived, she provided information that his
father has been increasingly forgetful for the last 12 months. Initially, he would forget where he placed
his keys, glasses. He would ask questions repeatedly, would forget names of friends and relatives.
This has progressed to difficulty finding his way even in familiar places. Neurologic exam shows an
awake, alert, elderly male who insists that he was just on his way to the mall. There were no moto-
sensory deficits. These manifestations are consistent with
A. Dementia
B. Delirium
C. Either dementia or delirium
D. None of the above

Kaplan 9th ed page 334-335

92. Which component of the neurologic exam would be most important in the diagnosis of this case
A. mental status exam
B. cranial nerve exam
C. motor exam
D. sensory exam

Kaplan 9th ed page 334-335

93. On further exam the patient was noted to be rigid with tremors and bradykinesia. What is your
consideration?
A. Cortical dementia
B. Subcortical dementia
C. Binswanger’s disease
D. Normal pressure hydrocephalus

Kaplan 9th ed page 334

94. What is the most common type of dementia


A. Alzheimer’s disease
B. Pick’s disease
C. Huntington’s disease
D. Vascular dementia

Kaplan 9th ed page 330

95. A 54 year old alcoholic was brought to the ER confused and disoriented with fluctuating
consciousness and perception. What is your diagnosis?
A. Alzheimer’s disease
B. Alcohol intoxication
C. Alcohol delirium
D. none of the above

Kaplan 9th ed page 405 - 406

96. A few months later the patient mentioned above came back because of memory impairment and
confabulation. What is your diagnosis?
A. Alzheimer’s disease

50
INTERNAL MEDICINE BOARD EXAM QUESTIONS
B. Korsakoff’s syndrome
C. Wernicke’s encephalopathy
D. None of the above

Kaplan 9th ed page 347

97. The following are reversible causes of dementia EXCEPT


A. hypothyroidism
B. vit b deficiency
C. normal presuure hydrocephalus
D. Creuztdfeldt-Jakob disease

Kaplan 9th ed page 364

98. Triad of Normal Pressure Hydrocephalus


A. gait disturbance,aphasia, memory disturbance
B. cerebellar signs, sensory signs, ocular signs
C. gait disturbance, urinary incontinence and dementia
D. memory impairment, ataxia and urinary incontinence

Kaplan 9th ed page 254

99. A 50 year old man was brought to the ER because of worsening confusion and distractibility. The
wife can’t pinpoint the exact onset of symptoms but remembers her husband to be acting different
some 2 months ago. Lately the patient has been complaining of headache.What’s the least likely
possibility?
A. Dementia
B. Delirium
C. Mood disorder
D. Primary psychotic disorder

Kaplan 9th ed page 360 - 361

100. On further exam the patient has lapses in memory and mild right facial weakness. What
diagnostic tool will you employ?
A. lumbar puncture
B. electroencephalograph
C. Ct scan of the head with contrast
D. None of the above

Kaplan 9th ed page 360 - 361

***END***

DEPARTMENT OF INTERNAL MEDICINE

_B____1. All of the following are emergencies and require immediate attention and
treatment, EXCEPT:
a. acute left ventricular failure c. ventricular fibrillation
b. acute rheumatic carditis d. status asthmaticus

_A_____2. A 16 years old woman came to you with complaint of chest pain. After work-
up you diagnose her to have mitral valve prolapse. You explain to her that MVP tends to
be benign but the following may occur, EXCEPT:
a. angiodysplasia c. sudden cardiac death
51
INTERNAL MEDICINE BOARD EXAM QUESTIONS
b. endocarditis infective d. embolic phenomena

_D_____3. A patient has a suspicious myocardial infarction 4 hours ago. The most
reliable test is:
a. SGPT c. SLDH
b. WBC d. CK-MB

_C_____4. The microorganisms associated with coronary heart disease are:


a. Chlamydia pneumoniae and Streptococcus pneumoniae
b. Chlamydia pneumoniae, Streptococcus pneumoniae and H. pylori
c. Chlamydia pneumoniae and Helicobacter pylori
d. Streptococcus pneumoniae and Helicobacter pylori

_B_____5. A poverty related heart disease:


a. congenital heart disease c. coronary artery disease
b. rheumatic heart disease d. cor pulmonale

_A_____6. Cardiac rehabilitation post-MI can be started


a. within 24 hours after admission c. not necessary
b. after discharge from the ICU d. after discharge from the hospital

_A_____7. The single biggest risk factor in the causation of stroke:


a. hypertension c. obesity
b. cardiac disease d. none of these

_B_____8. Enlargement of the heart on plain PA view of a chest x ray is evidenced by a


minimal cardiothoracic index of:
a. less than 0.3 c. greater than 1.0
b. greater than 0.5 d. greater than 1.2

_A_____9. Elevation of serum K is most toxic to the:


a. myocardium c. liver
b. kidney d. brain

_A_____10. Ventricular gallop or third heart sound is characteristic of:


a. mitral regurgitation c. tricuspid regugitation
b. aortic stenosis d. mitral stenosis

_C_____11. A 36 year old female is hypertensive and diabetic. She came in because of
an erythematous rash across the bridge of the nose. Her BP was 180/119. Which of her
medicines should be withdrawn?
a. propranolol c. hydrazine
b. nifedipine d. euglucon
_A_____12. The most effective way to demonstrate the area of blood clot in deep
venous thrombosis is by:
a. Doppler ultrasonography c. plain x ray
b. contrast venography d. impedance plethysmography

_B_____13. The usual cause of macrocytic anemia is:


a. chronic liver disease c. chronic renal disease
b. malabsorption d. iron deficiency

_A_____14. Steppage gait is caused by:


a. bilateral pyramidal tract disease c. exaggerated lumbar lord
b. double athetosis d. bilateral foot drop

_D_____15. Bell’s palsy is caused by damage to:


a.vagus nerve c. oculomotor nerve
52
INTERNAL MEDICINE BOARD EXAM QUESTIONS
b. trigeminal nerve d. facial nerve

_C_____16. Antiplatelet agent used in preventing further TIA are the following,EXCEPT:
a. dipyridamole c. etodolac
b. sulfinpyrazone d. aspirin

_C______17. The diagnosis of epilepsy is is best established by:


a. EEG c. history
b.CT scan d. physical examination

_A______18. The following are the cardinal signs of Parkinsonism, EXCEPT:


a. brisk reflexes
c. tremor at rest, impaired postural instability
b. bradykinesia
d. rigidity

_A_____19. The neurotransmitter deficient in Parkinsonism is


a. dopamine c. melatonin
b. serotonin d. norepinephrIne

_B_____20. Which is NOT a feature of Parkinson’s disease?


a. expressionless facies c. festinating gait
b. athetosis d. cogwheel rigidity

_D_____21. Basal skull fracture commonly involved in head injury:


a. temporal lobe c. frontal lobe
b. parietal lobe d. occipital lobe

_B_____22. The initial drug of choice in trigeminal neuralgia (tic doloreux) is


a. morphine c. digitalis
b. carbamazepine d. aspirin

_D_____23. Very small pupils that improve with Maloxone are characteristics of
a. transtentorial hemiation
c. marked increased intracranial pressure
b. severe anoxia
d. narcotic overdose

_B_____24. The following features are seen in delirium tremens, EXCEPT:


a. hallucination c. fever
b. alkalosis d. hypotension

_A_____25. Cognitive effect of stress:


a. all of these c. difficulty in concentration
b. short attention span d. tired of thinking

_A_____26. NOT a characteristic of infantile autism:


a. thought and perceptual
c. occurs before 36 months disturbances predominate
d. abnormal non-verbal communication
b. subnormal level of intelligence

_A_____27. One of the following is NOT feature of psychogenic depression:


a. flat effect c. anorexia
b. diarrhea d. lack of libido

53
INTERNAL MEDICINE BOARD EXAM QUESTIONS
_D_____28. The diagnosis of endogenous depression is bolstered by one of the following
laboratory tests:
a. serum cholinesterase c. electroencephalography
b. serum serotonin d. dexamethasone suppression

_B_____29. Lithium carbonate is the drug of choice for


a. hysteria c. depression
b. mania d. schizophrenia

_C_____30. Which goal is most important to develop a sense of intimacy?


a. finding a sexual partner c. finding a peer group
b. resolution of identity d. resolving moral issues

_D_____31. NOT a mature defense:


a. humor c. sublimation
b. ascetism d. suppression

_C_____32. Type of schizophrenia which shows less regression of mental faculties,


emotional response and behavior:
a. disorganized c. catatonic
b. undifferentiated d. residual

_B_____33. Fear of death at this age is because of the fear of abandonment:


a. 10-14 years c. 1-1/2 years
b. 2-1/2 to 5 years d. 1-1/2 to 2-1/2 years

_C_____34. Suicide is most commonly seen in


a. petit mal epilepsy c. affective disorders
b. schizophrenic reaction d. character disorder

_A_____35. A definitive contraindication to electroconvulsive therapy (ECT) is


a. brain tumor c. aortic aneurysm
b. threatened retinal detachment d. myocardial disease

_A_____36. The most common form of dementia in the elderly is


a. Alzheimer’s disease c. brain tumores
b. atherosclerosis d. cretinism

_B_____37. Definitive therapy of sleep apnea:


a. electrophrenic stimulation c. respiratory stimulants
b. tracheostomy d. nocturnal oxygen inhalation

_B_____38. A secretion is TENACIOUS if it


a. is not clear c. smells like decaying matter
b. is sticky and not easily pulled d. has salty taste
apart

_D_____39. Pathognomonic radiologic sign of pneumothorax:


a. tram line c. hyperinflation
b. hyperluscency d. visceral pleural line

_D_____40. In what condition is the lung ventilated but not perfused?


a. emphysema c. viral pneumonitis
b. atelectasis d. embolism

_A_____41. A respiratory consequence not found in pulmonary embolism:


54
INTERNAL MEDICINE BOARD EXAM QUESTIONS
a. fetor hepaticus c. pulmonary infarction
b. respiratory alkalosis and tachypnea d. pneumoconstriction

_B_____42. Bronchiectasis is commonly a sequel to


a. carcinoma of the lung c. pulmonary embolism
b. infection d. heart failure

_C_____43. Total lung capacity is usually increased with


a. congestive failure c. emphysema
b. cystic fibrosis of pancreas d. hyaline membrane disease

_A_____44. The honeycombed density seen on chest x-ray is characteristics of


a. bronchiectasis c. acute pulmonary edema
b. cardiac asthma d. pulmonary TB in a diabetic

_A_____45. Which most closely simulate miliary TB?


a. histoplasmosis c. chlamydia
b. candidiasis d. mycoplasma

_D_____46. Lung abscess is a complicating factor in comatose patients. This is due to


a. decubitus ulcers c. thrombophlebitis
b. repeated convulsions d. aspirations

_D_____47. The term “coin lesion” is diagnosed because of


a. hemoptysis c. aspirated foreign body
b. bronchoscopy d. chest x-ray finding

-B_____48. The best screening tool available for peripherally located lung cancer is the
a. sputum cytology c. chest radiograph
b. bronchoscopy d. CT scan

_D_____49. In respiratory acidosis, one of the following is retained:


a. HCI c. oxyhemoglob
b. sulfates d. carbon dioxide

A_____50. A chronic smoker who develops primary lung malignancy is apt to have
a. small cell carcinoma c. epidermoid carcinoma
b. adenocarcinoma d. large cell carcinoma

_C_____51. Type of bronchogenic carcinoma which commonly produces endocrine


manifestations:
a. epidermoid Ca c. small cell carcinoma
b. squamos Ca d. Adenocarcinoma

_C_____52. The risk of a non-smoker married to a smoker is


a. increased for ischemic heart disease only
b. not increased in any significant amount
c. increased by approximately 25% for lung cancer and ischemic heart disease
d. increased for lung cancer only

_B_____53. The highly acidic gastric content is neutralized in the duodenum largely by
the
a. bile acids
c. hydroxyl groups from the ileum
b. bicarbonate from the pancreas
d. residual secretion of the esophageal goblet cell

55
INTERNAL MEDICINE BOARD EXAM QUESTIONS
_C_____54. Which specific electrolyte disturbance in the body fluid leads to clinical
disorders?
a. iron c. hydrogen
b. zinc d. potassium

_C_____55. Fat absorption in the intestines is facillated by


a. amylase c. bile salts
b. mucopolysaccharides d. mucin

_D_____56. There may be extra-gastric sources of gastrin, excessive secretion of which


may lead to peptic ulcer. The most common nongastric source of this gastrin is
a. gall bladder c. uterus
b. lung d. pancreas

_C_____57. Spasms of abdominal pain that are colicky are most likely due to
a. peritonitis c. obstructed hollow viscus
b. referred pain from the chest d. paralytic ileus

_C_____58. Which of the following is present in the large bowel?


a. intrinsic nerves of the intestines c. absorption of water
b. extrinsic nerves of the intestines d. succus entericus

_D_____59. Pneumoperitoneum is present when there is


a. duodenal ulcer c. salpingitis
b. gall bladder empyema d. ruptured bowel

_A_____60. The following pathogens are causative agents of enteric infections. Which of
the following has basent fecal leucocytes?
a. Vibrio cholera c. Campylobacter jejuni
b. diarrheogenic d. Shigella

_B_____61. Paralytic ileus is NOT pathologic in one of the following:


a. ruptured appendicitis c. bile peritonitis
b. soon after abdominal surgery d. hypokalemia

_D_____62. A rise in BUN without renal disease, renal ischemia, obstruction is most
likely to be due to
a. severe liver disease c. hemolytic anemia
b. eclampisa d. massive GI bleeding

_B_____63. After major abdominal surgery there is


a. gastric hyperacidity c. volvulus
b. transient ileus d. reflux esophagitis

_A_____64. For disorders in the GIT such as ulcers, diverticula, strictures, new growths
and motility disorders the test that is most useful is
a. x-ray c. MRI
b. CT scan d. ultrasound

_B_____65. Ascitic fluid in cirrhosis of the liver would be expected to show


a. more than 1000 white cells/cubic mm
c. hemorrhage
d. positive malignant cytology
b. protein less than 2.5%- transudate
56
INTERNAL MEDICINE BOARD EXAM QUESTIONS
_C_____66. Ruptured peptic ulcer with pneumoperitoneum is most easily and safely
diagnosed by
a. barium swallow c. plain x-ray of abdomen
b. ultrasound d. paracentecis of the abdomen

_A_____67. An upright film of the abdomen shows free air in the peritoneum. This
means
a. perforated viscus c. volvulus-bird beak
b. diabetic acidosis d. hyperkalemia

_B_____68. Colonoscopy compared to Barium enema as a diagnostic procedure for


colorectal cancer has
a. less cost
c. greater chance of misdiagnosis
b. greater chance of perforation
d. less chance of hemorrhage

_D_____69. In an upright scout film of the abdomen, a step-ladder appearance of the


bowel indicates
a. enterocolitis c. peritonitis
b. volvulus d. bowel obstruction

_B_____70. Which of the following esophageal disorders is best characterized by


dysphagia for both solids and liquids?
a. Schatzki’s rings c. benign esophageal stricture
b. achalasia d. esophageal carcinoma

_D_____71. The fungus that frequently invades the upper GIT and causes esophagitis is
the
a. Blastomyces c. Cryptococcus
b. Aspergillus d. Candida

_D_____72. Lower esophageal sphincter (LCS) dysfunction is not uncommon. Which of


the following is useful in evaluation LES pressure?
a. endoscopy c. Barrium swallow
b. acid reflex test d. esophageal manometry

_A_____73. Repeated severe epigastric pain 1 hour after heavy meals in a 40-year old
obese female is usually due to
a. gall stones c. peptic ulcer
b. pancreatitis d. meteorism

_B_____74. Stress ulcer that occurs with burns, brain tumors and shock present
commonly as
a. hematochezia c. severe abdominal pain
b. melena d. intestinal obstruction

_B_____75. In the classification of polyps of the colon, what is/are considered


precancerous?
a. lymphosarcomatous c. Peutz-Jegners syndrome
b. all of these are correct d. adenomatous

_D_____76. Most of the ammonia is formed in the


a. small intestines c. kidneys
b. lungs d. liver

_B_____77. Albumin is a protein that is synthesized in the


57
INTERNAL MEDICINE BOARD EXAM QUESTIONS
a. pancreas c. nephron
b. liver d. lymphoid tissue

_C_____78. The urea cycle converts a potentially toxic substance to a more innocuous
substance. The toxic material is
a. urea c. ammonia
b. uric acid d. ornithine

_D_____79. Ammonia is highly toxic. It is detoxified by its transformation in the liver


into
a. alpha-ketoglutarate c. methionine
b. uric acid d. none of the above

_D_____80. Ammonia is most toxic when


a. the gut is sterile c. low serum Na+
b. serum K is high d. blood pH is high

_C_____81. Which constitutes a major portion of the bile?


a. cholesterol c. water
b. bile salts d. bile acid
_D_____82. The synthesis of bile salts takes place in the
a. common bile duct c. pancreas
b. duodenum d. none of the above-gallbladder

_C_____83. Pruritus is most severe in jaundice due to


a. portal cirrhosis c. biliary cirrhosis
b. hepatic schistosomiasis d. amoebic liver abscess

_C_____84. A large liver is least expected in


a. acute malaria c. chronic portal cirrhosis- liver will shrink
b. typhoid fever d. chronic heart failure

_B_____85. Long-lasting marker of hepatitis B:


a. anti-HBo c. anti-H
b. HbsAg d. anti-HBe

_B_____86. A patient with complete biliary obstruction developed bleeding tendencies.


The diagnostic test of choice is
a. fibrinogen blood level c. platelet count
b. prothrombin time d. BUN

_C_____87. An elevated serum SPGT is probably due to


a. jaundice c. liver necrosis
b. cholestasis d. bleeding varices

_B_____88. Delta viral hepatitis can only occur together with


a. hepatitis A c. leptospirosis
b. hepatitis B d. hepatitis

_A_____89. Which of the following drugs will be less potent in the presence of severe
liver disease?
a. phenytoin c. tetracycline
b. imipramine d. lidocaine

_D_____90. The most common benign neoplasm of the liver


a. sarcoma c. adenoma
b. focal nodular hyperplasia d. hemangioma

58
INTERNAL MEDICINE BOARD EXAM QUESTIONS
_B_____91. Most common primary tumor that spread/s to the liver is/are
a. prostate-bone c. melanoma
b. gastrointestinal tumor- colon d. bone

_D_____92. Pancreatic lipase preferentially splits triglycerides to form free fatty acids
and
a. delta-monoglycerides c. glycerol
b. alpha-monoglycerides d. beta-monoglycerides

_A_____93. The least symptomatic stones in the billiary tree are located in
a. gallbladder c. cystic duct
b. neck of the gall bladder d. common bile duct

_B_____94. The following pancreatic enzymes are potentially harmful, EXCEPT:


a. lipase c. lysolecithinase
b. none of these d. elastase

_C_____95. Blood level that does NOT rise in acute renal failure:
a. creatinine c. sodium
b. potassium d. uric acid

_B_____96. Pain described as “renal colic” usually indicates


a. kidney stone c. bladder stone
b. ureteral stone d. pyelonephritis

_A_____97. Which is acquired from infected water vapor or moist air inhaled?
a. legionella c. pseudomonas
b. mononucleosis d. typhus

_C_____98. Amylase is NOT elevated in one of the following diseases:


a. choledocholithiasis c. penetrating peptic ulcer
b. acute gastritis d. mumps

_D_____99. Urobilinogen is produced in the


a. renal tubules c. red blood cell
b. liver d. intestinal mucosa

_D_____100. Conjugation of bilirubin is a function of the


a. Kupfer cells c. intestinal mucosa
b. erythrocytes d. none of the above

@@@END OF EXAM@@@

REFERENCES:

1. Harrison’s: Principles of Internal Medicine Edition 2002


2. Braunwald, Fauci Edition 2002
3. National Medical Study in Medicine, 8th Edition
4. Board Review Series in Medicine; 4th edition

59
INTERNAL MEDICINE BOARD EXAM QUESTIONS

Das könnte Ihnen auch gefallen